#34 Rosh Review

अब Quizwiz के साथ अपने होमवर्क और परीक्षाओं को एस करें!

Question: What is the treatment of choice for a carotid artery dissection?

Answer: Anticoagulation (heparin followed by warfarin). Rapid Review Carotid Artery Dissection Most common cause of stroke in patients < 45 years Connective tissue disorders, neck trauma Abrupt unilateral neck pain or headache around the eye/frontal area, partial ipsilateral Horner's syndrome (miosis/ptosis) MRI/MRA Anticoagulation

Question: What are the common etiologies of spinal subarachnoid hemorrhage?

Answer: Arteriovenous malformation, tumors and anticoagulation. Rapid Review Cauda Equina Syndrome Patient will be someone with a history of trauma, malignancy, epidural abscess or hematoma Complaining of acute onset of lower back pain with weakness and numbness PE will show urinary retention, saddle anesthesia, decreased rectal tone Diagnosis is made by MRI or CT myelogram Most commonly caused by a herniated disc Treatment is operative decompression

Question: At what age is a patient allowed to express his or her concerns to a physician without a family member or guardian present?

Answer: At any age.

Question: Why are MS patients at increased risk for urosepsis?

Answer: Autonomic dysfunction causes urinary retention with high residual volumes. Rapid Review Optic Neuritis Idiopathic > MS Unilateral vision loss Pain with EOM Afferent pupillary defect Negative exam or optic disk swelling MRI to r/o MS Steroids Bilateral internuclear ophthalmoplegia (eyes can't look at nose): pathognomonic for MS

Question: Why should children receiving chemotherapy with anthracyclines be restricted from competitive sports?

Answer: Cardiotoxic effects of the medication that can lead to heart failure.

Question: What is the only FDA approved treatment of hiccups?

Answer: Chlorpromazine. Rapid Review Hiccups A rare manifestation of serious pathology Rx: chlorpromazine, metoclopramide

Question: What bedside test is helpful in ruling out vascular claudication?

Answer: Ankle-brachial index. Rapid Review Spinal Stenosis Radicular pain Pain: ↑ walking downhill, ↓ walking uphill

Question: Which ligament is most commonly damaged in an inversion injury of the ankle?

Answer: Anterior talofibular ligament. Rapid Review Ankle Sprain Patient with a history of ankle inversion PE will show pain and swelling Imaging will show partial or complete tearing of ligaments Most commonly injured anterior talofibular ligament (ATFL) Treatment is RICE therapy Comments: Ottawa Rules to determine imaging

Question: What percentage of patient's have metastatic disease by the time they are diagnosed with renal cell carcinoma?

Answer: 25-30%. Rapid Review Renal Cell Carcinoma Most common risk factor: smoking Flank pain + flank mass + hematuria Nephrectomy

Question: What is the most common dose of dexamethasone (IM or IV) given for the treatment of croup?

Answer: A single dose of 0.6 mg/kg. Rapid Review Laryngotracheitis (Croup) Patient will be a non-toxic appearing child, 6 months to 3 years old Complaining of URI symptoms with barky, seal-like cough, inspiratory stridor, low-grade fever Labs will show steeple sign on PA view Most commonly caused by Parainfluenza virus Treatment is steroids, aerosolized epinephrine

Question: How do you make the definitive diagnosis for Stevens-Johnson syndrome?

Answer: A skin biopsy.

Question: What is Trousseau's sign of malignancy?

Answer: Superficial thrombophlebitis associated with cancer, namely pancreatic carcinoma. Rapid Review Deep Vein Thrombosis (DVT) Patient with a history of smoking, long distance travel, surgery, oral contraceptives use Complaining of unilateral leg edema, leg pain, tenderness and warmth PE will show positive Homan's sign Diagnosis is made by first ultrasound, Gold Standard: venography Most commonly caused by stasis, hypercoagulable state, trauma (Virchow's triad) Treatment is IV heparin and switch to warfarin Comments: Risk stratification by Well's criteria

Question: What is Dance's sign?

Answer: Considered pathognomonic for intussusception: a sausage-like mass in the RUQ representing the actual intussusceptum and an empty space in the RLQ representing the movement of the cecum out of its normal position. Rapid Review Intussusception (Telescoping Bowel) Patient will be a child 5 months - 3 years old Complaining of colicky abdominal pain, vomiting and bloody stools (currant jelly) Diagnosis is made by ultrasound ("target sign") Most commonly caused by a tumor or meckel's diverticulum Treatment is air/contrast enema

Question: What is the name of the rapid quick arterial pulse seen in aortic regurgitation?

Answer: Corrigan's pulse. Rapid Review Mitral Stenosis Most common cause: rheumatic heart disease Dyspnea on exertion, hemoptysis Opening snap, diastolic apical murmur Atrial fibrillation

Question: What is the treatment of multiple myeloma?

Answer: Depending on risk stratification, chemotherapy and autologous hematopoietic cell transplantation. Rapid Review Multiple Myeloma Patient will be elderly Complaining of back pain PE will show CRAB: hyperCalcemia, Renal insufficiency, Anemia, lytic Bone lesions/Back pain Labs will show monoclonal antibody spike X-ray will show lytic lesions Peripheral blood smear: Rouleaux formations Serum protein electrophoresis: M spike Protein electrophoresis urine analysis: Bence-Jones proteins Most commonly caused by single clone plasma cell malignancy

Question: Which type of personality disorder would be present in a patient who is shy, withdrawn, and strongly desires to avoid close relationships?

Answer: Desired avoidance of close relationships is typical of schizoid personality disorder.

Question: What is the treatment of choice for Rocky Mountain Spotted Fever in the pediatric population?

Answer: Doxycycline. Regardless of the side effects that doxycycline can cause in the pediatric population, the benefits far outweigh the risks. Rapid Review Rocky Mountain spotted fever (RMSF) Patient with a history of recently in the woods hiking or camping Complaining of abrupt onset of severe headache, photophobia, vomiting, diarrhea, and myalgia PE will show maculopapular eruption on the palms and soles Diagnosis is made by skin biopsy Most commonly caused by Rickettsia rickettsia Treatment is ALWAYS doxycycline, even in children

Question: What is Reynold's pentad?

Answer: Found in patients with cholangitis, it includes right upper quadrant pain, fever, jaundice, hypotension, and altered mental status.

Question: What is a common side affect of sulfonylureas when used in elderly patients?

Answer: Hypoglycemia. Rapid Review Diabetes Mellitus Type 2 ↑ Insulin resistance Obesity, family hx Candidiasis Blurry vision Hyperosmolar hyperglycemic state Lifestyle changes First line pharmaceutical rx: metformin

Question: What is a dependent pocket of pus seen in the anterior chamber called?

Answer: Hypopyon. Rapid Review Endophthalmitis Infection of anterior, posterior, vitreous chambers Trauma IV ABX, ophthalmology consultation, admission

Question: What are the expected laboratory results in a patient with primary ovarian failure?

Answer: Increased follicle-stimulating hormone, increased luteinizing hormone, decreased estradiol. Rapid Review Primary Amenorrhea Definition: lack of menarche by 16 with normal 2° sexual characteristics or by 14 with no 2° sexual characteristics Etiologies: Hypothalamic/pituitary Ovarian Uterine Pseudohermaphroditism Lab workup: FSH, LH, prolactin, TFTs, testosterone, hCG Rx: treat underlying cause

Question: What is the treatment of asbestosis?

Answer: Supportive care: oxygen, pulmonary rehabilitation, pneumococcal and influenza immunization and pulmonary function monitoring. Rapid Review Asbestosis Shipping, roofing, plumbing SOB + nonproductive cough + chronic hypoxia CXR: "ivory white" calcified pleural plaques Ferruginous bodies Bronchogenic carcinoma > mesothelioma From the roof, but affects the base (lower lobes)

Question: What two malignancies affect the bowel of HIV patients?

Answer: Kaposi's sarcoma and lymphoma. Rapid Review HIV Clinical course: exposure --> acute HIV syndrome --> seroconversion --> asymptomatic period- -> symptomatic period Dx: ELISA followed by western blot Dx tests become positive during seroconversion (3-12 weeks after exposure) Chronic watery diarrhea: Cryptosporidium White cottage cheese lesions: Candida Irremovable white lesions on lateral tongue: hairy leukoplakia (EBV) Pneumonia, CD4 <200: PCP TB: CD4 <200, may have negative CXR/PPD Ring-enhancing intracranial lesions + focal neurologic deficits: Toxoplasma gondii Ring-enhancing intracranial lesions + AMS: primary CNS lymphoma Meningitis, CD4 <100: Cryptococcus Focal neurologic deficits, nonenhancing white-matter lesions, CD4 <50: PML (JC virus) Retinitis, cotton-wool spots: CMV Dark purple skin/mouth nodules: Kaposi's sarcoma Cutaneous: HSV, zoster reactivation

Question: What antibiotic can cause a disulfiram effect?

Answer: Metronidazole. Rapid Review Achilles Tendon Rupture Patient will be a deconditioned athlete With a history of fluoroquinolone use Complaining of "pop" or "snap" and sudden pain in the calf area PE will show absent plantarflexion upon calf squeeze (Thompson test) Treatment is posterior splint in plantarflexion, orthopedic consult

Question: What underlying process should be suspected if preeclampsia develops in the first trimester of pregnancy?

Answer: Molar pregnancy.

Question: What is the most common acquired neurologic disability in young adults?

Answer: Multiple sclerosis. Rapid Review Multiple Sclerosis Demyelinating disorder Caucasian females Optic neuritis (afferent pupillary defect, pain with eye movement, monocular vision loss, pale optic disc) Internuclear opthalmoplegia Lhermitte phenomenon: spinal electric shock sensation with neck flexion CSF: ↑ IgG protein, WBC pleocytosis Rx: methylprednisolone

Question: Patients with severe aortic regurgitation may benefit from which long-acting vasodilators?

Answer: Nifedipine XL or other long acting calcium channel blockers. Rapid Review Aortic Regurgitation Acute MCC: endocarditis Chronic MCC: rheumatic heart disease Blowing diastolic murmur at left sternal border Pulse pressure: normal (acute), widened (chronic) de Musset sign: head bobbing with systole Quincke's pulse: prominent nail pulsations Duroziez murmur: "singsong" murmur over femoral artery Austin-Flint murmur: mid-diastolic murmur in severe AR Rx objective: ↓ afterload

Question: Can staph scalded skin syndrome be spread to other body parts by rupture of the bullae?

Answer: No, the fluid inside the bullae is sterile. The toxin is produced at a remote site and delivered to the skin via the bloodstream. Rapid Review Staphylococcal Scalded Skin Syndrome (SSSS) S. aureus exotoxin < 5 years old Erythema → exfoliation → desquamation Superficial layers of epidermis shed Bullae with Nikolsky's sign ß-lactamase/penicillinase resistant PCN Children with eczema at higher risk

Question: What is another name for procaine?

Answer: Novocain, a common local anesthetic used by dentists—and an ester. Rapid Review Local Anesthetics Amides: lidocaine, bupivacaine (2 Is) Esters: tetracaine, benzocaine (1 I) Esters: allergenic 2º to (PABA) Benzocaine: methemoglobinemia Lidocaine: seizures, hypotension Bupivicaine: cardiotoxicity Concentration 1% = 1g/100mL = 10 mg/mL Toxicity Rx: lipid emulsion

Question: What are 3 predisposing factors for intertrigo?

Answer: Obesity, diabetes mellitus and HIV.

Question: What is the most common type of atrial septal defect?

Answer: Ostium secundum (75% of all cases), in which there is incomplete adhesion between the foramen ovale flap and the septum secundum. Rapid Review Atrial Septal Defect PE will show wide, fixed split S2 Most commonly caused by persistent ostium secundum Comments: May lead to Eisenmengers syndrome

Question: What are the common causes of conductive hearing loss?

Answer: Otitis externa or media, cholesteatoma, otosclerosis, tympanic membrane perforation and foreign bodies. Rapid Review Hearing Loss Conductive Weber: localizes to affected ear Rinne: abnormal (BC > AC) Most commonly caused by otitis media, serous otitis, and cerumen impaction Sensorineural Weber: localizes to unaffected ear Rinne: normal (AC > BC) Most commonly caused by excessive noise exposure, drugs, and normal aging

Question: In what position should a patient be placed when attempting manual reduction of an inguinal hernia?

Answer: Patient should be placed in Trendelenberg position with leg externally rotated and flexed. Rapid Review Incarcerated Hernia Irreducible Uncompromised blood supply

Question: What is the first ECG change seen in hyperkalemia?

Answer: Peaked T waves. Rapid Review Hypokalemia Patient with a history of diuretics use, diarrhea, vomiting Complaining of weakness, hyporeflexia, cramping, paresthesias ECG will show U waves, T wave flattening, ST-depression, QT prolongation Treatment is potassium replacement along with magnesium

Question: What is the name of the condition associated with infected hair follicles that may result in abscess, along the natal cleft of the buttocks?

Answer: Pilonidal disease. Rapid Review Perirectal Abscess Perform careful rectal exam Dull achy pain in rectum Complication: fistula Operative drainage required

Question: What is the most common cause of amenorrhea and galactorrhea in a premenopausal woman?

Answer: Pregnancy. Rapid Review Growth Hormone Deficiency Most common cause: pituitary tumor ↓ Muscle mass ↓ Bone density ↑ Lipids ↓ Memory ↓ IGF-1

Question: What conditions commonly cause false positive PPD results?

Answer: Prior infection with nontuberculous mycobacteria and a history of a bacilli Calmette-Guerin (BCG) vaccination Rapid Review Latent Tuberculosis Positive PPD criteria: 15 mm: no ↑ risk 10 mm: high-risk, homeless, health-care workers, IVDU, foreign born 5 mm: immunosuppressed, recent TB contact, abnormal CXR, steroid use ​Rx: INH for 9 months

Question: Pseudogout is often confused with which disease process secondary to its multi-joint presentation?

Answer: Rheumatoid arthritis, both have multiple joint involvement with symmetric distribution. Rapid Review Pseudogout (Chondrocalcinosis) M=F, >50 years old Slower onset than gout Positively birefringent, rhomboid-shaped, calcium pyrophosphate crystals Knee Rx: NSAIDs

Question: What is the unique pathogen that is associated with urinary tract infections in adolescent females?

Answer: Staphylococcus saprophyticus.

Question: What are the guidelines for the Human papillomavirus vaccine in males?

Answer: The Advisory Committee on Immunization Practices (ACIP) recommends the routine use of quadrivalent or 9-valent HPV vaccine in males aged 11 or 12 years. Beyond The Boards HPV Vaccination: Routine vaccination at age 11 or 12 years has been recommended by the Advisory Committee on Immunization Practices (ACIP) since 2006 for females and since 2011 for males. New recommendations support the use of a 2-dose schedule for girls and boys who initiate the vaccination series at ages 9 through 14 years. Three doses remain recommended for persons who initiate the vaccination series at ages 15 through 26 years and for immunocompromised persons.

Question: When should the nail be removed in a patient with a subungal hematoma?

Answer: The nail may be removed if it is split, avulsed or a laceration extends to the skin. Rapid Review Trephination, Nail Indication: acute, not spontaneously draining, intact nail folds, painful Methods: handheld electrocautery (preferred), heated paper clip, 18-gauge needle/syringe Post procedure care: soak in warm soapy water TID x 7 days

Question: Complicated acute diverticulitis can lead to fistula formation most commonly with which organ?

Answer: The urinary bladder. Rapid Review Diverticulitis Patient will be complaining of abdominal pain that is localized to the left lower quadrant, fever, nausea, vomiting, and a change in bowel habits PE will show localized guarding, rigidity, and rebound tenderness Diagnosis is made by CT scan Treatment is abx

Question: How long can button batteries remain lodged in the esophagus before they require removal?

Answer: There is no safe time frame for button batteries to remain lodged in the esophagus. Button batteries can cause liquefaction necrosis and should be removed immediately by endoscopy if there is any indication of impaction. Rapid Review Esophageal Foreign Body Site of obstruction: C6 > T4 > T11 AP/PA view: Flat side of coin appears Esophageal necrosis --> perforation. Most FBs that pass pylorus pass spontaneously Observe most esophageal FBs for 24 hours Emergent endoscopy indicated if FB is battery, sharp, or signs of obstruction present

Question: Which sex chromosome anomaly is suspected in men with persistent gynecomastia and small testes, particularly in those with language-based learning difficulties?

Answer: These symptoms are characteristic of Klinefelter syndrome in which men are typically born with a karyotype of 47,X,X,Y.

Question: True or false: the majority of patients with primary biliary cirrhosis are women?

Answer: True.

Question: What are some examples of dairy products that are best tolerated by patients with lactose intolerance due to their lower lactose contents?

Answer: Unpasteurized yogurt and aged cheeses are usually better tolerated than milk, cottage cheese, or ice cream, which contain much higher quantities of lactose. Rapid Review Lactose Intolerance Genetic condition or 2° to underlying conditions Dx: usually clinical, lactose tolerance test, lactose breath hydrogen test Rx: lactose restriction, calcium, vitamin D

Question: How often do patients with placental abruption present without pain or vaginal bleeding?

Answer: Up to 1/5th of patients with placental abruption will have neither bleeding nor pain. Rapid Review Placental Abruption Patient will be a pregnant women in her third trimester With a history of hypertension, trauma or cocaine use Complaining of painful vaginal bleeding Labs will show hypofibrinogenemia

Question: What stain is used to visualize Bartonella in a biopsy specimen?

Answer: Warthin-Starry stain. Rapid Review Cat Scratch Disease Patient with a history of scratch by a kitten or cat Complaining of subacute, regional lymphadenitis PE will show a vesicle, which became an erythematous papule Most commonly caused by Bartonella henselae Treatment is mainly supportive, azithromycin if needed

Question: Is the risk of epilepsy increased in children who have had a febrile seizure?

Answer: Yes, slightly. Risk is 1-2% instead of 0.5-1% in the general population. Rapid Review Febrile Seizure - Simple Patient will be a child 6 mos - 5 yrs Complaining of a single generalized seizure lasting < 15 mins Most commonly caused by rapid rise in temperature Treatment is supportive care

Question: Can steroids be started for temporal arteritis prior to obtaining a biopsy?

Answer: Yes. The biopsy should be performed within 48 hours of starting steroids. Rapid Review Temporal Arteritis (Giant Cell Arteritis) Patient will be a woman > 50 y/o Complaining of monocular visual loss, unilateral headache, jaw claudication PE will show tender temporal Artery Labs will show ESR > 50 Diagnosis is made by temporal artery biopsy Treatment is high dose steroids ASAP Comments: Associated polymyalgia rheumatica

Which of the following antibiotics is associated with spontaneous tendon rupture? Amoxicillin Doxycycline Levofloxacin Sulfamethoxazole

Correct Answer ( C ) Explanation: Fluoroquinolone drugs, including levofloxacin, have been associated with spontaneous tendon ruptures. Fluroquinolones are a commonly prescribed antibiotic class. The class includes ciprofloxacin, moxifloxacin and levofloxacin. The fluoroquinolone class of drugs has a number of side effects, the most serious of which are prolongation of the QTc and spontaneous tendon rupture. Tendon rupture appears to be more common in older patients. The overall risk is between 0.1 - 0.4%. These drugs are discouraged for use in pregnant women and children secondary to their effect on cartilage. Amoxicillin (A), doxycycline (B) and sulfamethoxazole (D) are not associated with an increased rate of spontaneous tendon rupture.

Which of the following scenario is most appropriate to safely discharge home a two-year-old who has swallowed a coin? The child is not drooling The child is tolerating oral sips and the parents are reliable The coin is in the stomach The coin is oriented in the sagittal plane at the level of the clavicular heads

Correct Answer ( C ) Explanation: Coins are the most common pediatric foreign body ingestion. Complications are rare, around 2%, but can be severe. The narrowest part of the GI tract is found in the esophagus. Once a foreign body passes out of the esophagus and into the stomach, it rarely causes major difficulties because it is propelled through the remaining GI tract by peristalsis with expulsion in a few days. A child with an esophageal foreign body may not be drooling (A) and may be able to tolerate oral intake (B). However, as long as the coin is in the esophagus, it will have to be removed endoscopically before the patient is discharged home due to the increased risks of perforation and esophageal erosion. Coins oriented in the sagittal plane (D) on radiograph are likely to be in the trachea rather than the esophagus. Coins in the esophagus lie in the coronal position in almost all cases because the opening into the esophagus is much wider in this orientation.

Which of the following electrocardiogram findings is most consistent with a diagnosis of hypokalemia? Flattened P waves Osborn waves Shortened QT interval U waves

Correct Answer ( D ) Explanation: Hypokalemia is defined as a potassium level < 3.5 mEq/L and is the most frequently seen electrolyte abnormality encountered in clinical practice. While most patients with hypokalemia are asymptomatic, severe potassium depletion can result in cardiac dysrhythmias and neurologic dysfunction (e.g. paresthesias, muscle weakness, and decreased deep tendon reflexes). Patients with known cardiovascular disease are at increased risk for conduction abnormalities related to hypokalemia. The likelihood of being symptomatic not only depends on the extent of the depletion, but on the rate at which the potassium level decreases. Electrocardiogram changes in hypokalemia include flattened or inverted T waves, as well as U waves, seen as an upward deflection following the T wave. Hypokalemia can cause increased amplitude and width of the P wave, not flattened P waves (B). Osborn waves (C), a prominent deflection of the J point, can be seen in hypothermia. A prolonged QT interval, not a shortened QT interval (D), can be seen in hypokalemia.

A 47-year-old man presents with hiccups for three days. He is unable to stop them with any home remedies. His physical examination is benign. Which of the following is a potential cause of hiccups? Hypercalcemia Hyperkalemia Hypoglycemia Hyponatremia

Correct Answer ( D ) Explanation: Hiccups commonly begin secondary to gastric distention resulting from a number of causes including excess eating, carbonation, and aerophagia. Additionally, excess alcohol ingestion as well as an acute emotional stress or excitement may lead to hiccups. When hiccups become intractable, evaluation for a more significant cause is indicated. Within the central nervous system, pathologies that lead to inhibition of the normal hiccup reflex include vascular malformations, meningitis, encephalitis, structural lesions. Any lesions that cause irritation to the vagus or phrenic nerves may lead to hiccups (e.g. tumors, infection, foreign bodies). Within the gastrointestinal system, there are multiple possibilities involving the stomach, esophagus and pancreas. Additionally, any lesions that potentially irritate the diaphragm (pneumonia, effusions) can cause hiccups. Myocardial ischemia or infarction is a rare cause of hiccups. In the metabolic category, hyponatremia and hypocalcemia are two possible causes of hiccups. Hypercalcemia (A), hyperkalemia (B) and hypoglycemia (C) are not common causes of hiccups

Which of the following medical conditions excludes a child from participation in sports? Current fever History of HIV infection History of persistent asthma Known seizure disorder

Correct Answer ( A ) Explanation: According to the AAP, a child with a fever should be excluded from sports participation. A fever may indicate a pathologic medical condition that can cause increased resting metabolism, heart rate, heat storage and risk of heat illness, and decreased heat tolerance and exercise capacity. Also, fever can sometimes signify myocarditis or other dangerous conditions. The child can resume sports once the fever resolves and they feel well enough to play. For most chronic health conditions, children are encouraged to participate in athletics with medical clearance. Conditions that absolutely exclude children from sports participation include carditis, fever, and severe infectious diarrhea. Other conditions that require cardiology evaluation and most often exclude individuals from sports participation include hypertrophic cardiomyopathy, acute rheumatic fever with carditis, and Ehlers-Danlos syndrome. Athletes with HIV (B) may be allowed to participate as their health permits but skin lesions must be covered properly and athletic personnel should use universal precautions when handling bloody or body fluids. Certain sports such as wrestling and boxing may increase risk of viral transmission and so if the athlete has a detectable viral load they should be advised to avoid high-contact sports. Children with asthma (C) should be able to participate in sports with proper medication and education; however, scuba diving should be preceded with caution. Well-controlled seizure disorders (D) do not exclude children from sports participation; however, children with poorly controlled seizures need assessment for collision, contact, or limited-contact sports. Riflery, swimming, weightlifting, power lifting, and strength training should be avoided in these children.

A 72-year-old man presents with a painful red eye and visual loss worsening over the last 24 hours. He recently had cataract surgery. Examination of the eye reveals the image above. Which of the following is the most likely? Endophthalmitis Hyphema Uveitis Vitreous hemorrhage

Correct Answer ( A ) Explanation: Endophthalmitis is an infection involving the anterior, posterior and vitreous chambers of the eye. It results from trauma (blunt globe rupture, penetrating injury, foreign bodies) and also iatrogenically after ocular surgery like cataract repair. Patients complain of severe pain in the eye and visual impairment or loss. Examination of the eye reveals decreased visual acuity, injected conjunctiva, chemosis and haziness of the infected chambers. Infections are treated with both systemic and intraocular antibiotics. A hyphema (B) is blood in the anterior chamber usually caused by trauma. When the patient is in an upright position, blood will layer along the inferior aspect of the anterior chamber. As the hyphema increases in size, it elevates intraocular pressure. In some cases admission is warranted for patients with large hyphemas (>50%), decreased vision, sickle cell disease and elevated intraocular pressure. Treatment is aimed at decreasing pressure with topical (beta-blocker, alpha agonist or carbonic anhydrase inhibitors) and systemic therapy (carbonic anhydrase inhibitor, mannitol). Uveitis (C) occurs after blunt trauma in which the iris and ciliary body are inflamed causing ciliary spasm. Patients complain of significant photophobia with significant eye pain. Examination of the eye reveals perilimbal conjunctival injection (also called ciliary flush) and a small, poorly dilating pupil. Photophobia occurs with light shone on both the affected and unaffected eye. On slit lamp, cells (white and red) and flare (protein) are noted in the anterior chamber. Treatment is with a topical cycloplegic agent to minimize spasm. Vitreous hemorrhage (D) occurs as a result of injuries to the retina, uveal tract and their associated vascular structures. Common associated conditions include diabetic retinopathy, retinal vein occlusion and trauma. Patients complain of decreased visual acuity and floaters. The condition is not typically painful. Diagnosis is made with ocular ultrasound showing blood products in the posterior chamber.

A 17-year-old presents for a well-child visit. She is healthy and has no complaints. A review of her past immunizations shows that she is up to date on all required immunizations at her 8-year-old well child visit. She also received the Human papillomavirus vaccine at age 15 and 16, and quadrivalent meningococcal vaccine and TdaP at age 12. Which one of the following vaccines should she receive at this visit? Human papillomavirus vaccine Inactivated poliovirus Measles Rubella

Correct Answer ( A ) Explanation: Human papillomaviruses (HPV) cause the most common sexually transmitted infection in the world and are responsible for nearly all cases of cervical cancer. Genital human papillomavirus infection can be divided into low-risk infections, causing genital warts, and high-risk infections, causing cervical intraepithelial neoplasia. The quadrivalent recombinant HPV vaccine (Gardasil®) protects against HPV types 6, 11, 16, and 18, and is approved for girls and women nine to 26 years of age. The bivalent HPV vaccine (Cervarix®) protects against types 16 and 18, but not types 6 and 11. It is approved for girls and women 10 to 25 years of age. Both types of the Human papillomavirus vaccine are given as a three-dose series in persons who initiated HPV vaccination at ages 15 through 26 years, so this patient is due for her third dose. The recommended interval between the first and third doses is 5 months, with approximately 3 months recommended between the second and third doses; however, the series can safely be completed at longer intervals. She is up to date on all immunizations at age 8, so it can be assumed that she has received her rubella (D), measles (C), and polio vaccinations (B).

You are about to repair a laceration on your patient when she tells you that she is allergic to lidocaine. Which of the following anesthetics is a suitable alternative? Bupivacaine Mepivacaine Prilocaine Procaine

Correct Answer ( D ) Explanation: Local anesthetics can be divided into two classes, the esters and the amides. The two most commonly used local anesthetics in the ED, lidocaine and bupivacaine, are amides. In a patient allergic to one class, a drug from the other class should be used. One way to remember which class each anesthetic belongs to is to use the following trick: all of the amides have two i's in their spelling, whereas the esters have only one i. Bupivacaine (A), mepivacaine (B), and prilocaine (C) are all amides.

On a Sunday afternoon, a surgical oncologist and his family attend a football game in the city where he practices. While at the game, he runs into a physician colleague that works at the same institution. After some casual small talk, his colleague inquires, "Are you taking care of Mr. Clarke, my personal trainer? I heard through the grapevine that he has melanoma, and I didn't know if you had started him on any chemotherapy or performed any surgical intervention yet. Hopefully you'll be able to take very good care of him." In this situation, the surgical oncologist may confirm which of the following? No information at all Only that Mr. Clarke is his patient The patient's diagnosis The patient's treatment plan

Correct Answer ( A ) Explanation: In order to be in compliance with patient confidentiality, a physician must not discuss any information regarding a patient's care with a physician who is not actively involved in that patient's care. This not only includes the diagnosis, treatment and prognosis of a patient's care, but also the confirmation or denial of whether or not a person is, in fact, a patient of the physician in question. A physician has an ethical responsibility to his or her patients to respect and protect their confidentiality in every situation, including non-physician interactions as well as with physician colleagues who are not involved in the active care of the patient. The most appropriate course of action in a situation as described above would be to withhold information concerning the patient's condition and medical course. Physicians should not be dishonest or lie in order to protect patient confidentiality. Federal and state laws designed to safeguard patient confidentiality are inadequate against the rapid and innovative use of electronic health websites. Health professionals must be aware that this information is not always secure and that they are accountable for maintaining privacy for the patient. A physician should not divulge any information about a patient, including names, diagnoses (C), treatment plan (D), prognoses, and care providers (B) to any other physician not currently taking care of that patient.

A 65-year-old woman who is morbidly obese presents to your office with intertrigo in the axilla. On examination you detect small, reddish-brown macules that are coalescing into larger patches with sharp borders. You suspect a secondary infection complicating the intertrigo. What is the most appropriate topical treatment for this condition? Erythromycin Mupirocin Nyastatin Zinc oxide

Correct Answer ( A ) Explanation: Intertrigo is a superficial inflammatory dermatitis occurring on two closely opposed skin surfaces as a result of moisture, friction, and lack of ventilation. Bodily secretions, including perspiration, urine, and feces, often exacerbate skin inflammation. Physical examination of skin folds reveals regions of erythema with peripheral scaling. Excessive friction and inflammation can cause skin breakdown and create an entry point for secondary fungal and bacterial infections, such as Candida, Group A beta-hemolytic streptococcus, and Corynebacterium minutissimum. Cutaneous erythrasma is caused by Corynebacterium minutissimum and presents as small reddish-brown macules that may coalesce into larger patches with sharp borders. They may be asymptomatic or pruritic and fluoresces coral-red on Wood lamp examination. Intertrigo complicated by erythrasma is treated with topical or oral erythromycin. In uncomplicated intertrigo, numerous agents and mechanisms can be used to keep the skin folds dry, clean, and cool. Applying barrier protectants reduces skin breakdown and alleviates pruritus and pain. Zinc oxide (D) can be used as a skin protectant to prevent further skin breakdown and introduction of secondary infections. Separating skin surfaces with absorbent products, such as gauze, cotton, and products with water vapor-permeable sheets, may also help reduce friction. Intertrigo complicated by fungal infection should be managed with topical antifungals. Nystatin (C) is effective only for candidal intertrigo. The diagnosis of secondary fungal infections is commonly made clinically, based on the characteristic appearance and distribution of satellite papules and pustules. The diagnosis may, however, be confirmed with a potassium hydroxide preparation positive for pseudohyphae and spores. Additionally, a potassium hydroxide preparation, Wood lamp examination, or culture of skin scrapings can diagnose conditions such as Candida or dermatophyte infections. The optimal treatment for patients with intertrigo complicated by group A beta-hemolytic streptococcal infections includes single or multiple regimens of topical therapies such as mupirocin (B).

Which of the following valvular disorders is characterized by any of the following: a low-pitched diastolic murmur heard best over the apex, an early high-pitched, blowing diastolic murmur heard best over the left sternal border, and a wide pulse pressure? Aortic regurgitation Aortic stenosis Mitral stenosis Tricuspid stenosis

Correct Answer ( A ) Explanation: Physical signs of aortic regurgitation (AR) include a rapid, quick arterial pulse (Corrigan's pulse), a wide pulse pressure, an early high-pitched, blowing diastolic murmur heard best over the left sternal border, an S3 gallop, and a low-pitched diastolic murmur at the apex (Austin-Flint murmur). Aortic valve regurgitation is defined as blood flow from the aorta to the left ventricle in diastole because of an incompetent aortic valve. Aortic valve insufficiency is generally acquired through valve infection, dilation and dissection of the aortic root, trauma, or long-term degenerative change of the valve, particularly in the setting of hypertension. Patients with a history of prosthetic valves can also have aortic valve insufficiency. Aortic insufficiency can also be caused by a congenital bicuspid aortic valve. The surgical treatment of AR is indicated in symptomatic patients with dyspnea, angina, or CHF. Asymptomatic patients should undergo surgery if left ventricular ejection fraction is 55% or less, or left ventricular end-systolic dimension approaches 5.5 cm. Patients with moderate to severe AR should avoid competitive sports, heavy workloads, and weightlifting. Murmur of Aortic Regurgitation 00:0000:00 The typical physical signs of severe aortic valve stenosis (B) are diminished carotid pulses (delayed and weak), a sustained apical impulse, a single second heart sound, an S4 gallop, and mid-systolic crescendo-decrescendo murmur late peaking best heard at the base of the heart. Most auscultatory signs of mitral stenosis (C) are missed if not performed in the left lateral decubitus position. Typically, the first heart sound (S1) is accentuated. A low-pitched diastolic rumble heard with the bell of the stethoscope over the apex is also present. The high-pitched opening snap, caused by the abrupt stopping of the domed mitral valve into the left ventricle is also appreciated in most patients midway between the left sternal border and apex. Tricuspid valve stenosis (D) is mostly caused by rheumatic heart disease and is typically associated with other valvular involvement. Patients can be dyspneic with activity. Typically, there is an increase in the jugular vein with a large a wave, indicating atrial contraction against a stiff tricuspid valve.

A 40-year-old man presents to the ED with the chief complaint of headache for two days. The headache is right frontal, constant, and severe. He has tried ibuprofen and acetaminophen without relief of the pain. He denies history of headaches, recent trauma, nausea, vomiting, and syncope. He has a past medical history of cervical disk protrusion for which he takes ibuprofen and gets manipulation by a chiropractor. His vital signs are T 36.6°C, BP 142/90, HR 82, RR 16, and oxygen saturation 99%. On examination, his right pupil is 3 mm and reactive; his left pupil is 6 mm and reactive. Extraocular movements are intact. He is unable to keep his right eyelid open against resistance. Visual acuity is 20/30 in both eyes with glasses. Fundoscopic exam is normal. His neck is supple. Strength is 5/5 in all extremities, gait is normal, and Romberg is negative. Which of the following is the most likely diagnosis? Carotid artery dissection Cluster headache Retinal artery occlusion Temporal arteritis Vertebral artery dissection

Correct Answer ( A ) Explanation: This patient has a carotid artery dissection. This is the most frequent cause of stroke in patients < 45 years old. Risk factors include minor neck trauma (cervical manipulation in this patient), family history of arterial disease, and connective tissue disorders. The most common presenting symptoms are unilateral neck pain or headache around the eye or frontal area. The pain is classically abrupt in onset. Carotid artery dissection is associated with a partial ipsilateral Horner's syndrome of miosis and ptosis. A carotid bruit may be heard on exam. The diagnostic study of choice is a MRI/MRA. If left untreated, it can lead to cerebral ischemia or retinal artery infarction and vision loss. Cluster headaches (B) are typically unilateral and characterized by severe orbital, supraorbital, or temporal pain lasting 15-180 minutes, and recurring throughout the day. It is also associated with ipsilateral red conjunctiva, lacrimation, nasal congestion, and rhinorrhea. Seventy percent of the time these headaches resolve with oxygen administration. Based on presenting headache features, distinguishing vertebral from carotid artery dissection can be difficult. However, dissection of the vertebral artery (E) is typically associated with marked occipital or posterior neck pain and may be accompanied by focal neurological deficits, including ipsilateral facial numbness and contralateral pain and temperature sensory loss. Retinal artery occlusion (C) is painless and is associated with loss of vision. Fundoscopic exam reveals a pale retina with cherry red macula. Temporal arteritis (D) tends to occur in patients over 50 years of age and involves severe, throbbing, frontotemporal pain and jaw claudication. On exam, there is tenderness or decreased pulse in the temporal artery. ESR measurement is a useful screening test, but the final diagnosis is made by arterial biopsy.

A 70-year-old man with a history of hypertension presents with complaints of fatigue and worsening lower back and pelvic pain. He denies history of trauma or fever. On examination, he is pale with diffuse tenderness to palpation of his lumbar spine and pelvis. Neurological examination is nonfocal. Laboratory studies show a normocytic anemia and a creatinine of 1.9 mg/dL. His X-ray is shown above. What is the most likely diagnosis? Cauda equina syndrome Lymphoma Metastatic prostate cancer Multiple myeloma

Correct Answer ( D ) Explanation: Multiple myeloma is a neoplastic proliferation of plasma cells in the bone marrow resulting in a monoclonal immunoglobulin. Occurring primarily in older adults, patients present with a range of symptoms best remembered by the mnemonic CRAB (elevated Calcium, Renal insufficiency, Anemia and Bone lesions). Calcium levels are elevated which can result in symptoms of renal colic, constipation, and confusion. Renal insufficiency is often the result of deposition of monoclonal light chains (Bence Jones proteins) in the renal tubules (also known as myeloma kidney). Hypercalcemia can also contribute to the renal insufficiency as well. Patients often have a normocytic, normochromic anemia with associated symptoms of fatigue, pallor, and generalized weakness. Lastly, osteopenia and extensive osteolytic lesions are commonly seen and may result in pathologic fractures. Patients experience bone pain primarily in the spine and trunk and less so in the extremities. Radiographs will show characteristic punched-out lytic lesions. Diagnosis is confirmed by finding > 10% clonal bone marrow plasma cells in addition to evidence of tissue or organ impairment as noted above. Symptoms of cauda equina syndrome (A) include lumbar back pain, variable motor and sensory changes in the lower extremities and bowel and bladder dysfunction (e.g. fecal incontinence, urinary retention with overflow incontinence). This is a surgical emergency. Primary lymphoma (B) of the bone is very rare. Bony metastases from lymphoma often cause single lytic lesions. Metastatic prostate cancer (D) is characterized by osteoblastic changes on radiograph, not osteolytic.

A 67-year-old woman presents with low back pain. She states that she has had pain for over 10 years but it has gotten worse recently. She also complains of pain shooting down her leg into her foot when she walks, which improves with bending forward. Which of the following disorders is affecting this patient? Spinal stenosis Spinal subarachnoid hemorrhage Transverse myelitis Vascular claudication

Correct Answer ( A ) Explanation: This patient's presentation is most consistent with spinal stenosis; a narrowing of the spinal canal leading to radicular symptoms on exertion. Spinal stenosis is most commonly seen in older patients (age >55 years). Typically, patients present with subacute back pain and lower extremity radiculopathies. The radicular pain occurs with walking and is relieved with rest thus mimicking claudication symptoms. Bending forward at the waist causes an increase in spinal canal diameter and reduces tension on the spinal cord leading to decrease in pain. Relief of pain with forward flexion at the waist is a unique finding in spinal stenosis. Sitting may also relieve the pain. Walking uphill may be painless (patient is flexed forward) while walking downhill (back extended) increases pain. Physical examination reveals lumbar radiculopathy (frequently of multiple levels) and increased pain with extension. Claudication (D) may be confused with spinal stenosis because pain is related to exertion but claudication will not be relieved by bending forward and should worsen with walking uphill. Spinal subarachnoid hemorrhage (B) typically results from an A-V malformation or tumor. Patients will have a sudden onset of severe pain as opposed to the chronic or subacute nature of spinal stenosis. Transverse myelitis (C) is an inflammatory process of the spinal cord that presents with combined motor and sensory deficits that may be at multiple levels.

A 19-year-old Asian man presents with complaints of diarrhea, flatulence, and bloating that occur everyday after breakfast. His breakfast is usually fruit with cottage cheese and a glass of milk. He denies weight loss, constipation, steatorrhea, bloody stools, or tenesmus. Which of the following tests will most likely diagnose his condition? Colonoscopy Hydrogen breath test IgA endomysial antibody test Qualitative fecal fat assay

Correct Answer ( B ) Explanation: A hydrogen breath test is the most commonly used test to diagnose lactase deficiency leading to lactose intolerance. Lactose intolerance is a very common condition worldwide, as most people of non-European heritage experience a reduction in the concentration of their lactase enzymes during childhood that continues to progress into adulthood. This results in varying manifestations of gastrointestinal distress when patients are presented with dairy products. While some patients may be able to tolerate a small quantity of lactose-containing products if consumed with other foods, others require complete avoidance to remain distress-free. Patients may complain of diarrhea, bloating, cramping abdominal pain, and flatulence after eating lactose products. However, signs of nutrient malabsorption, systemic illness, or weight loss should not occur with isolated lactose intolerance. These findings should alert clinicians to assess for other gastrointestinal disorders. If a hydrogen breath test results in a rise of breath hydrogen beyond baseline after consuming 50 grams of lactose, lactose intolerance is likely. Two week of abstaining from any lactose-containing products should then be recommended. Resolution of symptoms during that time is confirmative of lactose intolerance. At that point, patients should be counseled to seek replacements for the calcium generally found in dairy products to prevent osteoporosis. Lactase-containing preparations may be beneficial. A colonoscopy (A) does not help in diagnosing lactose intolerance, the most likely disorder in this patient. It would be more helpful if weight loss, tenesmus, or bloody stools were present to indicate an underlying malignancy or inflammatory bowel disorder needing structural assessment. An IgA endomysial antibody test (C), along with an IgA tissue transglutaminase antibody test, is considered a very sensitive and specific test to diagnose Celiac disease. However, this patients symptoms relative to dairy consumption made Celiac disease a comparably less-likely causative condition. A qualitative fecal fat assay (D) is valuable in patients suspected of conditions for which steatorrhea is a presenting symptom. Steatorrhea was not observed in this patient's history.

A 37-year-old woman presents with a one week history of increasing anal pain and swelling. The pain is throbbing and constant, even while not having a bowel movement. There is an area of swelling with erythema along a portion of the anus which is very tender to palpation on rectal exam. Which of the following is the most likely cause of this patient's condition? Anal fissure Anorectal abscess Anorectal fistula Pruritus ani

Correct Answer ( B ) Explanation: Anorectal abscess presents with throbbing, constant pain around the rectal area and is identified on exam by erythema and swelling around the anus. If the abscess is in the intersphinteric space, it may not be visible externally, though is usually palpable on digital rectal exam. An anorectal abscess is thought to arise from an infected anorectal gland. The treatment is incision and drainage; antibiotics are not indicated unless the patient is immunosuppressed or there is a surrounding cellulitis. About 40-50% of patients with anorectal abscess go on to develop an anorectal fistula, which occurs when a connection forms between the infected perianal crypt gland and the perineum as the infection tracks through the perianal tissues. The mean age of patients presenting with anorectal abscess is 40, and it occurs in men more commonly than in women (2:1). Anal fissure (A) is a laceration along the surface of the anus below the dentate line, and often appears as a linear tear on exam. Symptoms include sharp, "knifelike" pain and bleeding with the passage of stool. A combination of trauma, ischemia, and increased anal pressure are thought to contribute to the development of an anal fissure. Anal fissures are treated with agents to soften the stool, as well as topical medications such as nitroglycerin or nifedipine to relieve pain and speed healing. Anorectal fistula (C) is a condition that often follows an anorectal abscess, though can also occur in patients with Crohn's disease, radiation proctitis, prior anal surgery, or infections such as HIV and malignancy. Anorectal fistula develops when a connection forms between a peri-anal crypt gland and the perineum. Diagnosis is clinical, though a fistula may not be visible without exam performed under anesthesia. Treatment is typically surgical and is guided by the type of fistula, of which there are four: intersphinteric, transsphinteric, suprasphinteric, and extrasphinteric. Pruritus ani (D) refers to severe itching of the anus and perianal skin that may be triggered by many factors including diarrhea, certain medical conditions, fecal incontinence, hemorrhoids, infections, and irritants such as clothing, drugs or moisture. Diagnosis occurs by examination of the perianal skin which may appear excoriated. Digital rectal exam is recommended to rule-out co-existing rectal conditions that may be triggering pruritus ani. Treatment includes elimination of the underlying disorder, avoidance of possible food triggers such as chocolate, coffee, tea and citrus, and short-term use of medications to help break the itch-scratch cycle such as anti-histamines, topical steroids and topical lidocaine.

Which of the following is the most common infectious etiology of diarrhea in patients with AIDS? Clostridium difficile Cryptosporidium Cytomegalovirus Mycobacterium avium complex

Correct Answer ( B ) Explanation: As the CD4 count drops, patients with AIDS are at risk for opportunistic infections of the gastrointestinal tract. Worldwide diarrhea is a source of significant morbidity and mortality of HIV-positive patients because of poor access to antiretroviral therapy. HIV infects and damages the gut-associated lymphoid tissue (GALT) which leads to frequent infection in those with poorly controlled disease and lower CD4 counts. Medications also cause diarrhea as a side effect. Additionally, patients with longstanding disease develop malabsorption syndromes from the chronic inflammatory changes of the intestinal tract. In patients with AIDS, cryptosporidium is a common cause of diarrhea when the CD4 count falls below 100 cells/mm3. Clostridium difficile (A) is a cause of diarrhea in AIDS patients with the appropriate risk factors. Cytomegalovirus (C) is a viral infection that may lead to colitis and is of particular concern in AIDS patients with rectal bleeding and tenesmus. Mycobacterium avium complex (D) is a rare cause of diarrhea and typically only occurs once the CD4 count has fallen below 50 cells/mm3.

A 29-year-old woman presents to the psychiatric clinic for a "lifetime" of intense anxiety not relieved by 12-weeks on fluoxetine. She reports low self-esteem, feeling "very uneasy" in social situations, and says she wishes she could make friends but worries that she'll say something "dumb" to people. Which of the following personality disorders does she most likely have? Antisocial personality disorder Avoidant personality disorder Dependent personality disorder Paranoid personality disorder

Correct Answer ( B ) Explanation: Avoidant personality disorder is characterized by this patient's symptoms of low self-esteem and desire for relationships which remains fruitless due to persistent fear of rejection. Other symptoms of avoidant personality disorder include a hypersensitivity to criticism and failure, often making patients unable to hold jobs and maintain relationships. Personality disorders in general are present in patients from the time of their childhood, and may have resulted in development of recurrent maladaptive behaviors to cope with impaired personality traits. When maladaptive coping skills eventually prove unable to allow the patient to navigate their responsibilities and relationships successfully, anxiety and depression often present. Without proper management, psychosis may even develop. The cornerstone of treatment for personality disorders includes regular behavior therapy to extinct inappropriate behaviors, and allow patients to recognize appropriate behaviors. Group psychological counseling is often helpful. Medical therapy is targeted at symptoms and can include antipsychotics, anxiolytics, and selective serotonin reuptake inhibitors. Antisocial personality disorder (A) typically manifests as pervasively selfish, callous, and impulsive actions. Patients may have frequent legal problems and demonstrate an inability to learn from negative experiences. Low self-esteem and fear of rejection are not prominent features. Dependent personality disorder (C) is characterized by low self-confidence. However this personality disorder centers more on extreme passivity in relationships and a need for others to make decisions for the patient. Paranoid personality disorder (D), while characterized by an overarching tendency to anxiety and hypersensitivity, usually causes patients to be abnormally suspicious, hyper-alert, and often defensive toward those around them.

A patient of yours with a history of diverticulosis develops acute diverticulitis. He is admitted to the hospital for inpatient care. He is successfully treated and discharged home. He follows up with you a few days later. Which of the following should you most likely recommend to him at this time? Colonoscopy within 3 days High fiber diet until 6 weeks of no symptoms Low fiber diet until 6 weeks of no symptoms Sigmoidoscopy within 3 days

Correct Answer ( C ) Explanation: Diverticulosis refers to herniations through the mucosa and submucosa of the colonic wall. It is associated with a low fiber diet. It occurs mainly in the sigmoid colon. Patients are usually asymptomatic, but 10-25% will develop diverticulitis and about 10% will develop hemorrhage. Diverticulitis stems from retention of food and bacteria in the herniations, which leads to fecaliths, obstruction, compromise of the mucosal blood supply, infection and perforation. This causes left lower quadrant pain, fever, nausea, constipation and vomiting. Abdominal computed tomography is the imaging modality of choice. Treatment may include outpatient antibiotics, but in more complicated cases (e.g. evidence of abscess or perforation), inpatient care is necessary with intravenous fluids, antibiotics, and possibly surgery. A low fiber diet is recommended after an episode of acute diverticulitis until the patient remains asymptomatic for 6 weeks. A low fiber diet is especially effective in cases of acute diverticulitis, because it helps reduce the frequency of stools and allows the affected portion of the colon to adequately heal. Colonoscopy (A) or sigmoidoscopy (D) are contraindicated in acute diverticulitis due to an increased risk of perforation. A high fiber diet (B) is a recommended preventative strategy for patients with diverticulosis. However, high fiber intake is held in the setting of acute diverticulitis until the patient has no symptoms for 6 weeks after resolution.

A mother brings her 11-month-old infant into a rural ED for inconsolable crying. You note the infant lying in the stretcher crying with his knees drawn to his chest. You perform a physical exam and leave the room to order blood work. When you return to the examination room, you note the infant now appears lethargic. An abdominal radiograph is obtained and interpreted by the radiologist as nonspecific. Which of the following diagnostic tests should be performed next? Abdominal CT scan Abdominal ultrasound Nasogastric tube lavage Upper GI series

Correct Answer ( B ) Explanation: In patients with suspected intussusception, ultrasound is a quick, noninvasive diagnostic modality commonly used for the diagnosis of intussusception. One study reported that the overall sensitivity and specificity of ultrasonography for detecting ileocolic intussusception was 97.9% and 97.8%, respectively. The authors concluded that ultrasonography should be used as a first-line examination for the assessment of possible pediatric intussusception. Ultrasonography eliminates the risk of exposure to ionizing radiation and can help to depict lead points and residual intussusceptions. It also helps to rule out other possible causes of abdominal pain. On the transverse ultrasound scan, the intussusception appears as a multilayered or wrapped complex mass; longitudinally, it appears as a tube within a tube. Abdominal CT scan (A) is rarely needed to diagnose intussusception and is associated with unnecessary radiation exposure. Nasogastric tube lavage (C) has no role in the management of intussusception. Some patients with intussusception, however, will require nasogastric tube decompression if there is evidence of significant gaseous distention. Upper GI series (D) is used in the diagnosis of pyloric stenosis secondary to delayed emptying from the stomach to the duodenum, due to hypertrophy of the muscle surrounding the pylorus.

Which of the following patients should be started on isoniazid therapy? 21-year-old woman with a 5 mm PPD reaction and a history of IV drug abuse 40-year-old homeless man with a 10 mm PPD reaction 51-year-old doctor with a 5 mm PPD reaction 8-year-old boy with an 8 mm PPD reaction

Correct Answer ( B ) Explanation: Patients who are medically underserved (homeless, correctional institution residents) should be treated with isoniazid (INH) if their PPD reaction is >10 mm. The tuberculin skin test or PPD is the best tool available for detection of latent tuberculosis (TB). The antigens present in the PPD induce a reaction in patients with latent TB. After placement of the antigen, the patient has the reaction interpreted by a healthcare provider at 48-72 hours. Treatment of latent TB is guided by the reaction as well as patient characteristics as seen in the table below: Children <4 years of age should be treated if their reaction is >10 mm (D). Patients with a history of IVDA (A) and health care workers (C) should be treated if their PPD is >10 mm.

An elderly woman presents to the clinic 2 weeks after a lengthy hospital stay. She had repair of a hip fracture which required prolonged bed rest. Her main complaint today is an area of "painful redness" on her leg. Examination reveals a nodular, rope-like, tender superficial vein on the anteromedial proximal thigh. You order a duplex ultrasound which reveals superficial vein thrombus with extension into the deep veins of the thigh. Which of the following is the most appropriate when considering anticoagulation in this patient? Aspirin Enoxaparin Ibuprofen Unfractionated heparin

Correct Answer ( B ) Explanation: Phlebitis is the counterpart to vasculitis, in that phlebitis is inflammation of a vein. Thrombophlebitis is vein inflammation due to thrombus (blood clot). These commonly occur in the superficial veins of the lower extremities, whereas, deep venous thrombosus (DVT) is the term used when a clot lodges in the deep veins. Symptoms include painful, palpable cord-like veins, distal edema and possibly localized erythema. Causes include a predilection to clotting, vein injury, incompetent venous valves (varicosity), Behcet's syndrome and prolonged travel or immobility. Evaluation usually includes Doppler ultrasonography and blood clotting studies. The key to treatment is driven by where the disorder is located. Below-the-knee superficial thrombophlebitis is usually treated with NSAIDs, compressive support and leg elevation. Above-the-knee superficial thrombophlebitis is commonly associated with deep venous thrombus. As such, dislodgement of the deeper clot can cause a life-threatening pulmonary embolus. Therefore, it is important to ascertain whether or not an above-the-knee superficial thrombophlebitis occurs with a deep venous clot (via Duplex ultrasound). If it does, then anticoagulation is recommended as part of the treatment plan. If it isn't, the recommended treatment is less clear. Enoxaparin is a low molecular weight heparin (LMWH) that is used to prevent and treat deep vein thrombosis or pulmonary embolism. It is administered as a subcutaneous injection and does not require aPTT monitoring which makes it ideal for outpatient treatment. Aspirin (A) inhibits platelet function but does does not have anticoagulant properties and is therefore not a recommended treatment for DVT. Ibuprofen (C) has a platelet inhibitory effect but is not an anticoagulant and is therefore not a recommended treatment for DVT. However, NSAIDs are the treatment of choice for superficial thrombophlebitis. Unfractionated heparin (D) is administered intravenously and requires monitoring of the aPTT every 6 hours which is not conducive for outpatient therapy. LMWH preparations offer more reliable anticoagulation, can be used in the outpatient setting, do not require aPTT monitoring and are associated with a lower risk of bleeding complications.

A patient complains of new onset hematuria, left flank pain and unintentional weight loss. Her past medical history is significant for chronic hypertension and tobacco use. Abdominopelvic imaging reveals a mass within the confines of the left renal capsule. A surgical biopsy is taken. When considering the most common kidney cancer in adults, the histology report of this patient's biopsy would most likely describe abnormalities of which of the following cells? Connective tissue cells of the renal stroma Epithelial cells of the proximal convoluted tubule Stromal cells of the metanephrogenic blastema Transitional cells of the renal pelvis

Correct Answer ( B ) Explanation: Renal cell carcinoma is the most common type of kidney cancer in adults. It originates from the epithelial cells of the proximal convoluted tubule. There are 5 different histological subtypes of renal cell carcinoma. The most common risk factors are tobacco use, obesity, prolonged anti-inflammatory use and hypertension. Most tumors remain clinically silent until advanced progression. Most renal cell carcinomas are found incidentally on unrelated imaging reports. Only 10% of cases present with the classic triad of flank pain, flank mass and hematuria. Other symptoms include weight loss, fever, hypertension, hypercalcemia, night sweats and malaise. The characteristic radiographic appearance of renal cell carcinoma is a solid, irregular or lobulated border, mass which distorts the renal contour. Initial treatment involves a radical or partial nephrectomy, as this cancer is relatively resistant to radiotherapy and chemotherapy. Some cases respond to immunotherapeutics. Cancers of the renal stromal connective tissue or blood vessel cells (A) are called renal sarcoma, which represent <1% of all kidney cancers. Stromal or epithelial cells of the metanephrogenic blastema (C) of a developing kidney are the source of Wilms tumor, the most common abdominal malignancy in children. Wilms tumor almost never occurs in adults. Most kidney cancers are renal cell carcinoma (80-95%), arising from the proximal convoluted tubule. The remaining 5-20% is mostly comprised of the second most common kidney cancer, urothelial cell carcinoma, which arises from the transitional cells of the renal pelvis (the junction between the ureter and kidney) (D).

A 4-year-old girl presents to the office with lymphadenitis of several left cervical lymph nodes. An erythematous papule is noted on the left hand. What is the treatment of choice? Amoxicillin-clavulanate Azithromycin Clindamycin Glucocorticoids

Correct Answer ( B ) Explanation: The above patient has a classic presentation of Cat Scratch disease caused by Bartonella henselae. Inoculation of Bartonella results from a scratch by a kitten or cat or via a flea bite. The scratch or bite often results in a vesicle, which classically transforms into an erythematous papule. The lymphadenitis caused by Bartonella may develop anywhere between five days and two months after inoculation, but the papule can often still be located on the extremity distal to the lymphadenitis. The most commonly affected lymph nodes are in the axillary region, followed by the cervical region. Lymphadenopathy is localized in 2/3 of patients but may involve multiple anatomic regions in 1/3 of patients. Generalized lymphadenopathy is rare. The treatment of choice for Cat Scratch disease is five days of azithromycin. However, most Cat Scratch disease will resolve in 1-4 months if untreated. Amoxicillin-clavulanate (A) is used to treat Staph, Strep, or anaerobic lymphadenitis in regions where methicillin-resistant Staphylococcus aureus is uncommon. Clindamycin (C) provides good coverage for Staph, Strep, and anaerobic lymphadenitis, although clindamycin-resistant Staph aureus is prevalent in some areas of the US. Clindamycin does not treat Bartonella. Glucocorticoids (D) may be used to decrease inflammation in the setting of lymphadenitis that is causing airway obstruction. However, steroids do not treat the infection that has caused the lymphadenitis. Of note, glucocorticoids can mask or delay a diagnosis of malignancy and may disqualify patients from some cancer protocols. Thus, glucocorticoids should not be used routinely in lymphadenitis and especially if the lymphadenopathy is suspicious for malignancy.

A 37-year-old man presents with worsening back pain for 1 week. He states that he has had back pain for years but over the last week, his pain has increased and he has weakness in his legs. Examination reveals 4/5 strength in the left leg and 3/5 strength in the right leg with decreased sensation. He has numbness to his buttocks and a post-void residual of 150 ml. What imaging study should be obtained? CT of the lumbar spine MRI of the lumbar spine Non-contrast head CT X-ray of the lumbar spine

Correct Answer ( B ) Explanation: This patient presents with signs and symptoms concerning for cauda equina syndrome and should emergently have an MRI done to determine the cause of compression. The syndrome results from compression of multiple lumbar and sacral nerve roots usually caused by a central disk herniation. It can also be caused by trauma, malignancy, epidural hematoma, or abscess. In cauda equina syndrome patients have a sudden onset of back pain and radiculopathies of multiple levels involving both legs. Changes in bowel or bladder function also commonly occur with urinary retention being the most consistent examination finding. As neurogenic bladder develops, patients will develop overflow incontinence. Urinary retention has a high sensitivity (90%) and negative predictive value (99.99%). Thus a low postvoid residual makes the disease highly unlikely, although it can not be used to definitively rule out the diagnosis. Other common findings are saddle anesthesia (75%) and decreased rectal tone (60-80%). MRI is the best modality for determining the cause as it has excellent tissue resolution particularly of the intervertebral disks and spinal cord. MRI also has the ability to detect more subtle soft tissue pathology as seen in epidural abscess. In patients who cannot get an MRI, CT myelogram is a reasonable alternative. CT of the lumbar spine (A) is useful in diagnosing acute fractures but has poor resolution of the spinal cord and intervertebral disks. Non-contrast head CT (C) is not helpful as the patient has a peripheral disorder. X-ray of the lumbar spine (D) has low utility, as it does not adequately visualize the spinal cord, intervertebral disks or other soft tissues.

A 25-year-old woman presents to the emergency room with a fever of 104°F complaining of chills, right back and side pain, as well as frequency and pain on urination. She admits to being sexually active for the first time the previous week, without the use of a condom. History is otherwise unremarkable. Physical examination is that of a well-developed and well-nourished young female in mild to moderate distress. Vital signs are pulse is 94, respiration is 24, blood pressure is 100/60, and temperature is 103.5°F. There is significant right CVA tenderness and tenderness over her bladder. Gynecologic exam demonstrates erythema of the vaginal and urethral mucous membranes and scant purulent urethral discharge. There is no abdominal, uterine, or fallopian pain nor pain on lateral motion of the cervix. What is the most likely diagnosis? Candida vaginitis E. coli urinary tract infection N. gonorrheae cervicitis Pelvic inflammatory disease

Correct Answer ( B ) Explanation: This young woman has an E. coli urinary tract infection (UTI). E. coli is the etiology of the majority of UTI's, reflecting the colonization of the lower gastrointestinal tract and the ascending nature of many UTI's. It is not unusual for the first UTI in a female to occur at the time that she is sexually active for the first time giving rise to the name "honeymoon cystitis." In this case, there is evidence that both the upper and lower urinary tracts are involved because of the high-grade fever, chills, and CVA pain as well as the elevated pulse and respiratory rates and the slightly low blood pressure. Candida vaginitis (A) presents with thick white, cottage cheese-like vaginal discharge and puritic denuded erythematous vaginal and vulvar mucous membranes. It is rarely associated with urinary tract infection but is, however, a common colonizer of the mouth and vagina. Gonorrhea (C) is a sexually transmitted infection caused by Neisseria gonorrhoeae. It is often asymptomatic in females or may be manifest with urethritis, endocervacitis, and salpingitis. It may progress to pelvic inflammatory disease (PID). It too is not associated with UTI signs and symptoms and tends to present with chronic rather than acute symptoms. Pelvic inflammatory disease (D) is a spectrum of inflammatory processes involving the female upper genital tract and caused by several different organisms. The most common manifestations are dull, continuous, lower abdominal pain, vaginal discharge and abnormal vaginal bleeding. Adnexal tenderness especially on lateral motion of the cervix is the most characteristic physical finding.

Which of the following findings best distinguishes cholangitis from acute cholecystitis? Elevated alkaline phosphatase Fever Jaundice Leukocytosis

Correct Answer ( C ) Explanation: Acute cholecystitis and cholangitis have many similarities in both presentation and laboratory findings. Both can present with right upper quadrant pain, nausea, vomiting, and fever. Patients with cholangitis tend to be more toxic in appearance with possible hypotension and altered sensorium. Laboratory studies for both can also include leukocytosis, elevated alkaline phosphatase, and moderately elevated aminotransferases. An elevated serum bilirubin level (and associated jaundice) is characteristic of cholangitis and infrequently seen in cholecystitis. This may serve to distinguish the two processes. Elevated alkaline phosphatase (A), leukocytosis (D), and presence of fever (B) can be found in both acute cholecystitis and cholangitis.

A 65-year-old man presents with cough and progressive dyspnea on exertion. He has no significant cardiac disease. He worked as an asbestos remover for thirty years. Examination reveals crackles but no wheezing. Pulmonary imaging reveals multiple basilar round opacities and pleural plaques. This patient is most at risk for developing which of the following diseases? Loffler's syndrome Lofgren's syndrome Mesothelioma Multiple myeloma

Correct Answer ( C ) Explanation: Asbestosis is one of the occupational lung diseases known as pneumoconiosis. It is a chronic inflammatory condition of the pulmonary interstitium, resulting in fibrosis and restrictive lung disease. It occurs in those chronically exposed to asbestos, namely manufacturers of asbestos-containing products or those involved in the removal of asbestos-containing materials. Symptoms usually only manifest after long exposure occurs. They include a progressive and sometimes severe dyspnea, especially on exertion, and dry cough. Bibasilar crackles are typical. In addition to the interstitial fibrosis, pleural thickening and plaques, as well as pleural effusion, occurs. Respiratory failure may be the end result of this condition. Pulmonary function testing reveals restrictive ventilatory defects. Diagnosis is mostly based on the classic occupational history and the key imaging findings of pleural plaques and small, basilar opacities. Although not necessary in most cases, biopsy results will show asbestos bodies. A rare form of cancer, malignant mesothelioma, is most commonly caused by chronic asbestos exposure. Although this cancer mainly occurs in the mesothelial cells of the pleura, it can also develop in the pericardium, peritoneum and tunica vaginalis. Loffler's syndrome (A) is a transient case of pulmonary infiltrates, fever, dyspnea, cough and eosinophilia in those infected with parasites, not those with asbestos exposure. Lofgren's syndrome (B) is defined as arthritis, hilar lymphadenopathy and erythema nodosum. Although it shares manifestations similar to sarcoidosis, another interstitial lung disease, it is not associated with asbestos exposure. Multiple myeloma (D) is a malignant transformation of the antibody-producing plasma cells, which manifests as anemia, recurrent infections, hypercalcemia, renal disease and bone pain. Asbestos exposure is not a risk factor in the development of this cancer.

Which of the following is a characteristic of pseudogout? Crystals are not seen with pseudogout Metatarsophalangeal (MTP) joint is most commonly affected Positive birefringent crystals Uric acid crystals

Correct Answer ( C ) Explanation: Calcium pyrophosphate deposition disease (CPDD) is known as pseudogout because of the acute gout-like attacks that CPPD crystals can cause. Chondrocalcinosis refers to radiographically detectable densities in cartilage and joint inflammation caused by these calcium-containing crystals. Calcium pyrophosphate (CP) crystals can be deposited not only on articular cartilage but also in ligaments, tendons, soft tissues, and synovium. Patients with pseudogout present with a monoarticular inflammatory arthropathy similar to gout. The knee is the most commonly affected joint. Arthrocentesis reveals weakly positive birefringent rhomboid-shaped crystals. Treatment of acute pseudogout consists of NSAIDs or corticosteroid injections. Colchicine can be used for chronic cases. Crystals (A) are present in pseudogout. They are composed of calcium pyrophosphate (D), not uric acid as seen in gout. The knee (B) is the most commonly affected joint in pseudogout. The MTP joint is most commonly affected in acute gout. The crystals of pseudogout are weakly positive birefringent rhomboid-shaped crystals, unlike the negatively birefringent needle-shaped crystals of gout.

A young woman reports 3 months of worsening vision, noting blurred and diplopic abnormalities. She also complains of left leg and right arm weakness. Examination reveals extraoccular palsies, poor visual acuity, nystagmus, left leg and right arm strength deficits and a positive Romberg sign. You suspect a central inflammatory demyelinating process. Which of the following imaging and results would help to confirm a diagnosis? Noncontrast head CT scan with cervical spine central stenosis Noncontrast head CT scan with ventricular enlargement Noncontrast T1-weighted brain MRI with corpus callosal hyperintensities Noncontrast T2-weighted brain MRI with periventricular hyperintensities

Correct Answer ( D ) Explanation: Multiple sclerosis (MS) is a chronic multifocal demyelinating neurologic disorder that involves different parts of the CNS at various points in time. There is a female to male ratio of 2:1. Prevalence increases with further distance from the equator. Clinically patients present with focal limb weakness, numbness, paresthesias, a band like sensation around the trunk or a limb, and ataxia. Diplopia, dysarthria, intention tremor, and bladder dysfunction are also seen. Optic neuritis (sudden loss or blurring of vision in one eye) is the presenting symptom in 25% of cases. Symptoms are transient, lasting days to weeks. Lhermitte sign (an electric shock-like sensation down the spine with flexing the neck) may be positive, with paresthesias in the trunk and limbs with neck flexion. An afferent pupillary defect may be seen. Diagnosis is usually made by MRI, which is the most sensitive study. The classic brain noncontrast MRI findings are multiple T2 hyperintensities found most commonly in the periventricular white matter, brainstem, cerebellum and spinal cord. CSF analysis on lumbar puncture may show oligoclonal bands. Treatment is with interferon-beta1b and 1a, glatiramer acetate and symptomatic therapy with corticosteroids. CT scan (A and B) is not the preferred imaging modality for diagnosing MS. Cervical spine stenosis may cause extremity weakness, but not cranial nerve palsies. Enlarged brain ventricles typically represent normal-pressure hydrocephalus in elderly patients. Common symptoms are dementia, cognitive deficits and urinary incontinence. Noncontrast T1-weighted MRI shows hypointensities, not hyperintensities (C), in patients with MS, commonly in the corpus callosum.

A 66-year-old man presents with difficulty hearing with the left ear. You begin a physical examination. He hears vibration more in the left ear during a Weber test. During Rhinne testing of the left ear, he reports hearing vibration better when the tuning fork was placed on the mastoid process than when it was placed next to the ear canal. You send him to the audiology lab. They report he has a 78 decibel loss in the left ear. Which of the following types of hearing impairment is present in this patient? Mild conductive Mild sensorineural Severe conductive Severe sensorineural

Correct Answer ( C ) Explanation: Hearing impairment is described as conductive or sensorineural. In conductive hearing impairment, there is a fundamental problem with the mechanical passage of sound waves from the external auditory canal through the tympanic membrane, ossicles, oval window, cochlear ducts, perilymph and endolymph. Comparatively, sensorineural hearing impairment occurs when pathology affects the cells of the organ of Corti, the vestibulocochlear nerve (cranial nerve 8), pons or brain. Two tests, the Weber and Rhinne tests, can assist in determining the type of unilateral hearing loss. During Weber testing, a vibrating tuning fork is placed on the skull's vertex. If the fork's vibrations are heard better in the affected ear, conductive impairment of the affected ear is present; if it is heard better in the normal ear, sensorineural impairment of the affected ear is present. During Rhinne testing, a vibrating tuning fork is placed on the mastoid process on the same side as the affected ear. When the patient reports he or she cannot hear it any longer, the fork is moved beside the external auditory canal to allow sound to pass thru the air. The patient is then asked to determine at which site they heard the sound better. If it was heard better when the fork was on the mastoid process, there is conductive impairment of the affected ear; if it was heard better through the air, sensorineural impairment is present in the affected ear. Hearing impairment can be further defined as mild, moderate and severe, based on the amount of loss measured in decibels. A deficit of 26-40 decibels is considered mild, while a loss of 41-70 decibels is considered moderate. Hearing loss >70 decibels is referred to as severe. During Weber testing, a vibrating tuning fork is placed on the skull's vertex. If the fork's vibrations are heard better in the affected ear, conductive impairment of the affected ear is present as in this case; if it is heard better in the normal ear, sensorineural impairment (B and D) of the affected ear is present. During Rhinne testing, if sound was heard better when the fork was on the mastoid process, there is conductive impairment of the affected ear as in this case; if it was heard better through the air, sensorineural impairment (B and D) is present in the affected ear. A deficit of 26-40 decibels is considered mild (A and B). This patient's loss is >70 decibels and is considered severe.

A 14-year-old girl has been diagnosed with primary amenorrhea. Pelvic ultrasound reveals an absent uterus. Genetic testing confirms a 46,XX karyotype. Which of the following is the most likely diagnosis? Asherman's syndrome Imperforate hymen Mullerian dysgenesis Turner's syndrome

Correct Answer ( C ) Explanation: Mullerian dysgenesis (normal complement of sex chromosomes: 46,XX) is a uterine abnormality in which there is a congenital absence of the uterus and vaginal upper two-thirds. Primary amenorrhea can be a result of several gynecologic anatomic abnormalities as defined in the chart below. Vaginal abnormalities: vaginal agenesis, in which the vagina does not develop; transverse vaginal septum, in which there is a failed fusion of the portions of the vagina; and imperforate hymen (B), in which there is a persistence of the urogenital membrane. Asherman's syndrome (A), in which past uterine procedures, like cesarean section or dilation and curettage, causes formation of uterine synechiae. Ovarian abnormalities: The most common ovarian cause of amenorrhea is primary ovarian failure due to gonadal dysgenesis/agenesis. The abnormal ovaries in these patients do not produce estrogens, a lack of which results in increased gonadotropin levels which is the condition known as hypergonadotrophic hypogonadism. Amenorrhea caused by gonadal dysgenesis/agenesis is usually due to sex chromosome abnormalities, such as Turner's syndrome (45,XO) (D), which cause a premature depletion of primordial follicles and subsequent estrogen deficiency.

The typical flow of blood across an atrial septal defect occurs in which of the following fashions? Deoxygenated blood from left to right atria Deoxygenated blood from right to left anterior descending coronary arteries Oxygenated blood from left to right atria Oxygenated blood from right to left atria

Correct Answer ( C ) Explanation: One of the congenital cardiac abnormalities is atrial septal defect (ASD). This condition is characterized by a defect in the interatrial septum. This condition initially results in a left-to-right shunt as oxygenated blood from the higher-pressure left atrium passes into the right atrium. This congenital defect can go undiagnosed for 40-50 years, as it is typically asymptomatic. In fact, the majority of cases (70%) are diagnosed in the fifth decade. Over time, a large defect may lead to pulmonary overcirculation, pulmonary hypertension, Eisenmenger syndrome, right-to-left shunting, and cyanosis. When symptomatic, patients complain of fatigability, exertional dyspnea or recurrent respiratory infections. During childhood, many atrial septal defects will spontaneously close, however, ASDs that persist into adulthood usually require surgical closure. The classic finding on cardiac auscultation is a fixed, split S2. Atrial septal defects do not affect the coronary arteries (B). The left atrium does not contain deoxygenated blood (A) as it has just returned from the lungs via the pulmonary veins. The right atrium does not contain oxygenated blood (D) as it has just returned from the periphery via the inferior and superior vena cava.

A 21-year-old woman G1P0 at 35 weeks gestation presents with headache, blurry vision, and shortness of breath. Vital signs include a blood pressure of 195/110 mm Hg, heart rate of 90 beats per minute, respiratory rate of 21 breaths per minute, oral temperature of 37.1℃, and oxygen saturation of 90% on room air. Urinalysis reveals 3+ protein. A chest radiograph reveals pulmonary edema. Which of the following medications should be administered to reduce blood pressure? Enoxaparin Furosemide Hydralazine Magnesium sulfate

Correct Answer ( C ) Explanation: The patient has severe preeclampsia. Preeclampsia refers to the constellation of hypertension after 20 weeks of pregnancy (blood pressure ≥ 140 mm Hg systolic or ≥ 90 mm Hg diastolic) plus proteinuria (> 0.3 g in a 24-hour collection period). Severe preeclampsia refers to marked hypertension with evidence of end-organ dysfunction. Diagnostic criteria include two or more of the following: blood pressure ≥ 160 mm Hg systolic or ≥ 110 mm Hg diastolic (measured on two occasions at least four to six hours apart), visual disturbances, mental status changes, pulmonary edema, epigastric or right upper quadrant pain, elevated liver function tests, thrombocytopenia, proteinuria, oliguria, or impaired fetal growth. If a patient with preeclampsia develops seizures, it is termed eclampsia. Clues in this scenario pointing to severe preeclampsia include third trimester pregnancy with hypertension, proteinuria, visual disturbance, and pulmonary edema. The pathophysiology of preeclampsia is not fully understood but likely involves increased vascular reactivity and vasoconstriction to the placenta and other maternal organs. Delivery of the infant is the definitive treatment. Mild preeclampsia can be monitored without treatment, but severe preeclampsia should be treated with antihypertensives and magnesium for seizure prevention until delivery can be safely achieved. Antihypertensives of choice for severe preeclampsia include intravenous labetalol, hydralazine, and nifedipine. Patients presenting to the ED with signs and symptoms of preeclampsia should always be co-managed with an obstetric consultant. Enoxaparin (A) is an anticoagulant used in the treatment of venous thromboembolic disease. Although pulmonary embolism is on the differential for a pregnant patient with shortness of breath and hypoxia, preeclampsia is more likely in this clinical scenario. Furosemide (B) (a loop diuretic) is not first line in the treatment of preeclampsia since the pathophysiology involves vasoconstriction and hemoconcentration, not fluid overload. Magnesium sulfate (D) is used for seizure prevention in preeclampsia and for the treatment of eclampsia, but it is not specifically an antihypertensive.

A 2-year-old boy presents to the ED with painful swelling in the scrotum for the past 4 hours. His parents report a bulge in the groin that comes and goes but now is not going away. On exam, you note engorgement of the right scrotum. The child screams in pain when you try to touch the mass. Which of the following is the most appropriate next step in management? CT scan Manual detorsion Morphine sulfate Urinalysis

Correct Answer ( C ) Explanation: The patient in this case has an incarcerated inguinal hernia. A hernia is a protrusion of peritoneal contents through a defect in the abdominal wall. Incarcerated hernias result from bowel becoming edematous and engorged, with the inability to be reduced. A strangulated hernia occurs when there is vascular compromise of the herniated contents. Inguinal hernias are due to a patent processus vaginalis. Incarcerated hernias are a surgical emergency. If there are no signs of strangulation or systemic toxicity, manual reduction should immediately be attempted. Adequate analgesia and sedation should be provided prior to attempting reduction. The patient in this clinical scenario is clearly in pain and should be administered analgesia—morphine sulfate. This will aid in performing the physical exam and allow an attempt at manual reduction. A CT scan (A) is not indicated in this case. Ultrasound is the diagnostic test of choice to help discern herniation of bowel from other causes of scrotal masses in infants. Manual detorsion (B) is a critical maneuver to perform in suspected testicular torsion. Torsion can present with a tender, swollen, and enlarged scrotum (similar to the appearance of an incarcerated inguinal hernia), but it is not associated with a mass that appears and disappears. Urinalysis (D) is not immediately helpful in this patient and should not delay treatment.

A 16-year-old girl presents with pain to the right index finger after slamming it in a car door. Physical examination reveals a 50% subungal hematoma and an X-ray shows a non-displaced distal tuft fracture. What management is indicated? Oral antibiotics, splinting and follow up Splinting and follow up Trephination, splinting and follow up Trephination, splinting, oral antibiotics and follow up

Correct Answer ( C ) Explanation: This patient should have trephination performed to relieve pain from the subungal hematoma, be splinted in extension and follow up with a hand surgeon. Subungal hematomas are common after crush injuries to the digits. They are often associated with distal phalanx fractures. Pain associated with these hematomas can be severe as pressure increases under the nail. Management focuses on diagnosing associated injuries and providing pain relief. The easiest way to relieve pain in the digit is by relieving the pressure building under the nail. This can be accomplished with trephination of the nail. Typically, an 18-gauge needle or a paper clip (after straightening) is heated and then placed over the center of the hematoma. Gentle pressure is applied until the nail is penetrated. This typically results in a drop or two of blood exiting through the hole that has been created. The patient typically experiences immediate pain relief. Nail removal is typically unnecessary. Administration of antibiotics (A) has not been shown to be helpful to prevent infection. Splinting and follow up (B) are appropriate after trephination. Even though trephination in the presence of a distal phalanx fracture technically converts a closed fracture into an open one, it is not contraindicated to perform drainage of the blood. Prophylactic antibiotics (D) are not necessary. As usual, tetanus prophylaxis should be administered as needed.

A 30-year-old man presents with progressive memory impairment and mental fatigue after sustaining head trauma during a motorcycle accident. He also reports significant weight gain even though his diet is unchanged. Physical exam reveals generalized muscle weakness and a loss of muscle bulk but no dry mucous membranes or peripheral edema. A basic metabolic panel is normal. Electrodiagnostic testing shows no evidence of myopathy or neuropathy. A DEXA scan uncovers the presence of low bone mineral density. Which of the following serum laboratory tests would help you in securing a diagnosis? Antidiuretic hormone Fasting glucose Growth hormone Insulin-like growth hormone-1

Correct Answer ( D ) Explanation: A childhood deficiency of growth hormone (GH) causes dwarfism. However, adults may also be diagnosed with growth hormone deficiency. Growth hormone is normally produced and secreted by the pituitary gland, a process which is stimulated by hypothalamic growth hormone releasing-hormone and inhibited by somatostatin. Symptoms of growth hormone deficiency in the adult include decreased muscle mass, weakness, poor exercise tolerance, decreased bone density, increased subcutaneous fat and weight gain, poor cognition, mental fatigue and memory impairment. Some patients will display manifestations of other pituitary hormone deficiencies. Causes are congenital, acquired (head trauma or radiation) and idiopathic, with the most common cause being a pituitary tumor. It is diagnosed by a low level of serum insulin-like growth factor-1, an insulin tolerance test or a GHRH-arginine test. Antidiuretic hormone (A) abnormalities are considered in the evaluation of total body water derangements, such as polyuria and sodium disorders. This patient displays no symptoms of dehydration or fluid overload, and his sodium level (BMP) is normal. Fasting glucose (B) is used in the diagnosis of diabetes mellitus. It would not elucidate a cause of weakness and muscle atrophy. Random serum GH (C) levels are useless in the work-up due to its pulsatile release.

A 3-year-old boy is brought to the urgent care clinic by his mother. She tells you he has had a fever and cough for the past 2 days. His temperature is 38.6°C (101.5°F) and his respiratory rate is 35/min. Auscultation of the lungs reveals inspiratory stridor with a prolonged inspiratory phase. A chest X-ray shows subglottic narrowing. Which of the following is most likely the best treatment for this child's condition? Antibiotics Anticholinergics Bronchodilators Corticosteroids

Correct Answer ( D ) Explanation: Based on the constellation of findings, this patient most likely has croup (laryngotracheobronchitis). Croup typically presents with the classic triad of barking cough, inspiratory stridor, and hoarseness. It is most often caused by a viral infection (parainfluenza virus). Outpatient treatment usually consists of oral steroids (eg, oral dexamethasone) or nebulized epinephrine for severe cases. Corticosteroids are beneficial due to their anti-inflammatory actions. Since laryngeal edema is commonly seen in patients with croup, corticosteroids are extremely effective in decreasing this inflammatory process. Improvement is typically seen within 6 hours after administration of oral steroids. Since most cases of croup are caused by viruses, the use of antibiotics (A) is not indicated. Antibiotics should only be used to treat specific bacterial complications. Anticholinergics (B) (eg, tiotropium, ipratropium bromide) are long-acting bronchodilators used in the management of chronic obstructive pulmonary disease (COPD). Anticholinergics are not used in the treatment of croup. Bronchodilators (C) are typically used in the treatment (and management) of a variety of obstructive lung diseases (eg, asthma, COPD). They are not indicated in the treatment of croup.

A 35-year old woman presents to your office with a 1-week history of high fevers, severe headaches and muscle pain. She also reports a rash. She denies any recent travel or changes in her diet. She is current on her immunizations. She currently lives in North Carolina and enjoys hiking in the outdoors. On physical exam her temperature is 102.3°F and she has a maculopapular rash on her extremities, including her hands and feet and sparing her face. She states the rash began a few days after her fever started and has progressively worsened. You decide to draw laboratory tests and titers in order to confirm the suspected disease. You also prescribe antibiotics immediately. Which of the following is most likely etiology of the disease? Babesiosis Ehrlichiosis Lyme disease Rocky Mountain spotted fever

Correct Answer ( D ) Explanation: Based on the history and presentation, this patient most likely has Rocky Mountain Spotted Fever (RMSF) which is a tick-borne disease caused by the organism Rickettsia rickettsia. The organism is endemic in parts of North, Central, and South America, especially in the southeastern and south-central United States, such as North Carolina. History of hiking in the outdoors increases the likelihood even though there is no specific history of a tick bite. RMSF should be considered in patients with unexplained febrile illness even if they have no history of a tick bite or travel to an endemic area. The classic clinical triad of fever, headache, and rash should raise a high suspicion for RMSF, especially high fevers over 102°F. The rash begins as a maculopapular eruption on the wrists and ankles and spreads centripetally to involve the trunk and extremities. The face is usually spared. After exposure to vector ticks, patients who develop fever, petechial rash, and vomiting require antibiotic therapy. Antibiotic therapy should be initiated before laboratory confirmation is available. Doxycycline is the antibiotic of choice for RMSF. Doxycycline therapy also treats Lyme disease, ehrlichiosis, and relapsing fever; diseases often clinically confused with RMSF. Doxycycline should be initialed immediately in this patient. Babesiosis (A) is a tick borne illness that presents with Influenza-like symptoms, fever, sweating, myalgia, headache, hemolytic anemia, hemoglobinuria, jaundice, renal failure. Ehrlichiosis (B) is a tick-borne illness that presents with influenza-like syndrome, fever, chills, cough, malaise, headache, and myalgias. A rash is rare in this disease, differentiating this from RMSF. Lyme disease (C) is a tick borne illness that initially presents as an erythema migrans rash at the bite site, influenza-like symptoms, fever, fatigue, arthralgias, headache, cough, and lymphadenopathy. A bull-eye lesion is pathognomonic for Lyme disease.

A 22-month-old whose immunizations are up to date presents after a generalized seizure. Mom reports one seizure lasting less than five minutes. She has had rhinorrhea for the last two days and has a temperature of 102°F. On exam, she is well appearing and has returned to baseline. Which of the following is the most appropriate plan? Antiepileptic therapy CBC and blood culture Lumbar puncture Supportive care

Correct Answer ( D ) Explanation: Febrile seizures are defined as the presence of seizure in a febrile child without CNS infection. They are a common cause of seizure in children and most commonly occur between 6 months and 5 years of age with a peak occurrence between 14 and 18 months. The risk of seizure is correlated more with the rapid rise in temperature rather than an absolute number. A simple febrile seizure is brief, single, and nonfocal or generalized. Complex febrile seizures are prolonged, recurrent or focal. Evaluation of the simple febrile seizure is dictated by the evaluation of the fever and not the seizure. Patients should receive supportive care for the post-ictal state and their underlying illness. The use of antipyretics does not decrease the frequency or recurrence of seizures. CBC and blood culture (B) is not indicated in this patient. In the fully vaccinated child aged 3 months to 36 months, routine laboratory testing is not part of diagnostic algorithm. A urinalysis and culture is considered in females less than 24 months. Lumbar puncture (C) is suggested by the AAP for children with the following criteria: signs of meningeal irritation, incompletely immunized, or those with prior antibiotic therapy. It is also more strongly considered in children with complex seizures or those less than 12 months. Antiepileptic therapy (A) has no role after a single febrile seizure nor does referral for neuroimaging or EEG monitoring.

A 42-year-old woman presents to your office with complaints of fatigue, pruritus and abdominal discomfort. Laboratory testing reveals elevated serum alkaline phosphatase and hyperlipidemia. Physical exam findings include jaundice, skin excoriations and hepatomegaly. Which of the following is the most likely diagnosis? Acute pancreatitis Pancreatic cancer Polycythemia vera Primary biliary cirrhosis

Correct Answer ( D ) Explanation: Primary biliary cirrhosis is a rare disease of the liver that causes progressive cholestasis and can result in end-stage liver disease. It is believed to be autoimmune in nature, with T-lymphocytes attacking the bile duct epithelial cells, leading to their destruction. The loss of these bile ducts causes the signs and symptoms of cholestasis. Patients may be asymptomatic or present with fatigue, pruritus and right upper quadrant discomfort. Physical exam findings include skin hyperpigmentation, excoriations, jaundice, and hepatosplenomegaly. Physical manifestations of liver cirrhosis such as spider nevi, ascites and muscle wasting may be seen in later stages of the disease. Laboratory abnormalities include elevated alkaline phosphatase, antimitochondrial antibodies, antinuclear antibodies and hyperlipidemia. Diagnosis is determined when the patient has no comorbidities affecting the liver, no extrahepatic biliary obstruction and has at least two of the following: evidence on histology, the presence of antimitochondiral antibodies, and elevated alkaline phosphatase at least 1.5 times the normal limit. Treatment goals are to alleviate symptoms and slow progression of the disease. The only life-saving procedure available is liver transplantation. Patients with acute pancreatitis (A) experience severe abdominal pain, nausea and vomiting. Laboratory findings include elevated serum amylase and lipase. Patients with pancreatic cancer (B) may present with fatigue, weight loss, abdominal pain, jaundice and hepatomegaly. Pruritus is not commonly seen in pancreatic cancer. Patients with symptoms suspicious for pancreatic cancer should have initial diagnostic imaging done with transabdominal ultrasound. Polycythemia vera (C) causes aquagenic pruritus, burning pain in hands and feet, venous or arterial thromboses, transient visual disturbances and gastrointestinal complaints. Laboratory testing results include elevated hemoglobin, hematocrit, red blood cell mass and vitamin B12.

Which of the following anti-diabetic drug's mechanism of action works through the incretin system? Glipizide (Glucotrol®) Metformin (Glucophage®) Pioglitazone (Actos®) Sitaglipitn (Januvia®)

Correct Answer ( D ) Explanation: Sitaglipitn (Januvia®) is a DDP4 inhibitor that works through the incretin system. The naturally occurring incretins, glucagon-like peptide (GLP) and glucoinsulinotropic peptide (GIP), enhance beta-cell insulin secretion after glucose ingestion. The incretin effect is the increase in insulin secretion after gastrointestinal (GI) absorption of glucose compared to the amount of insulin secreted in response to glucose infused intravenously. The augmentation of insulin secretion is secondary to a stimulatory effect of GLP and GIP on beta cells. The metabolic clearance of these hormones is governed by dipeptidyl peptidase (DPP), which enzymatically inactivates the incretins. Thus, inhibition of DPP by drugs known as gliptins causes a more sustained postprandial insulin secretion. Glipizide (A) is a sulfonylurea that acts by shutting down membrane potassium ion channels, allowing membrane calcium channels to open, thereby establishing a calcium gradient that stimulates insulin secretion in beta cells. Metformin (B) suppresses glucose production by the liver, which reduces insulin resistance. It helps reduce LDL cholesterol and triglyceride levels, and is not associated with weight gain. Pioglitzaone (C) is a thiazolidinediones (TZDs) and was developed as transcription factors that stimulate the induction of many cellular proteins facilitating insulin action in the cytosol and mitochondria and works on insulin resistance and not the incretin system.

A 52-year-old man with a history of hypertension presents with a concern that he's "developing breasts." On exam, glandular enlargement beneath his areola is present bilaterally in a symmetric manner. It is mildly tender. Which of the following daily medications most likely contributed to his condition? Clonidine Hydrochlorothiazide Lisinopril Spironolactone

Correct Answer ( D ) Explanation: Spironolactone is well-established as a contributor to the development of gynecomastia due to its effect on androgen expression. Many medications are implemented in causing gynecomastia, which common offenders being antipsychotics, antifungal medications, high active antiretroviral therapy, and some atypical antidepressants. Fortunately gynecomastia caused by medication use usually resolves with discontinuation of the causative medication. If a medication is not found to be responsible for gynecomastia, other causes should be pursued. Serum beta-hCG levels should be drawn to look for a testicular tumor or other malignancy that may be producing exogenous hormones. Similarly, a prolactin level should be checked to rule out a prolactinoma. Additionally, any male athlete should be asked about taking anabolic steroids as using these will lead to gynecomastia in 50% of users. Other causes to explore based on individual patient profiles may include liver disease, obesity, and primary or secondary hypogonadism. Of note, in teenagers a mild degree of pubertal gynecomastia can be normal and will usually resolve without treatment in 1-2 years. However, any male breast tissue that is asymmetric, unusually firm, forming in a location other than below the areola, or accompanied by nipple bleeding, discharge, or retraction should raise concern for a malignancy. In these patients, a needle biopsy with cytologic examination will help differentiate benign gynecomastia from a tumor or other inflammatory process. When necessary, true glandular gynecomastia usually responds to selective estrogen receptor modulators, such as raloxifene or tamoxifen. Surgical correction may be needed in patients for whom medication therapies are ineffective. Clonidine (A), hydrochlorothiazide (B), and lisinopril (C) are incorrect choices as these are not particularly well-known for causing gynecomastia in patients.

A 50-year-old man currently on a regimen of penicillin develops a diffuse rash and facial swelling. On physical exam there are diffuse dark reddish purple papular lesions on his trunk, face, and extremities with extensive blister formation. Prior to the onset of the rash the man complained of a nonproductive cough, fever, and chills. Based on his history and physical exam, what is the most likely diagnosis? Herpes zoster Impetigo Scabies Stevens-Johnson syndrome

Correct Answer ( D ) Explanation: Stevens-Johnson syndrome (SJS) is a hypersensitivity reaction of the skin and mucous membranes due to medication, malignancy, or a medical condition. Drugs and malignancy are most often implicated as the etiology in adults and the elderly whereas infections are more often the cause in children. It begins with non-specific upper respiratory infection symptoms and then progresses to dermal manifestations. The rash begins as painful macules and papules which progress to purpuric and vesicular lesions. They rupture and cause extensive sloughing. It can be life-threatening if not treated immediately and adequately with antibiotics, steroids, pain medications, and IV fluid hydration. Debridement and skin grafting may also be necessary. Herpes zoster (A) is caused by a herpesvirus which affects the nerves. Patients develop a rash that follows a dermatome. The rash begins as painful macules that progress to vesicles on an erythematous base. It occurs due to reactivation of varicella virus which remains latent in the central nervous system. The duration of infection may decrease with anti-viral medication. Scabies (C) is a contagious condition caused by a mite that burrows underneath the skin causing a pruritic papular rash. Permethrin cream is a commonly used treatment. It is uncomfortable, but not life-threatening. Impetigo (B) is a contagious skin condition that most commonly occurs in children. It is commonly caused by either staphylococcus or streptococcus infections. It begins as blisters with cloudy yellow fluid that rupture causing red raised lesions with a honey-yellow crust. Mupirocin is a commonly used treatment. It is not a life-threatening condition.

A 62-year-old woman presents complaining of ankle pain following a fall from standing onto the ice last night. She is able to walk across the emergency department and back, but states that the pain becomes unbearable toward the end. Her posterior lateral malleolus and her anterior medial malleolus are tender to palpation. Which of the following is the correct indication for ankle radiography in this patient? Age Inability to bear weight after several steps Tenderness over the anterior medial malleolus Tenderness over the posterior lateral malleolus

Correct Answer ( D ) Explanation: The Ottawa Ankle Rule is a clinical decision making rule that was developed to allow clinicians to effectively select patients who are candidates for ankle radiography after acute ankle trauma. It consists of five criteria: tenderness over the posterior lateral malleolus, tenderness over the posterior medial malleolus, tenderness over the navicular bone, tenderness over the base of the fifth metatarsal, and inability to bear weight for four steps. If any one of the criteria are present, the patient is a candidate for plain films of the ankle. Age (A) is not a criterion for ankle radiography according to the Ottawa Ankle Rule, although an age of 55 years or greater is a criterion for knee radiography according to the Ottawa Knee Rule. Inability to bear weight after several steps (B) and tenderness over the anterior medial malleolus (C) are not criteria included in the Ottawa Ankle Rule.

A 55-year-old woman presents to the office with progressive dyspnea, paroxysmal dyspnea, orthopnea, and fatigue over the last several months. On auscultation of her heart you hear a low-pitched diastolic rumble best heard in the left lateral decubitus position along with a high-pitched opening snap. Which type of valvular abnormality is associated with these findings? Aortic regurgitaion Aortic stenosis Mitral regurgitation Mitral stenosis

Correct Answer ( D ) Explanation: The main symptoms of mitral stenosis (MS) are slowly progressive dyspnea and fatigue. Most auscultatory signs of MS are missed if not performed in the left lateral decubitus position. Typically, the first heart sound (S1) is accentuated. A low-pitched diastolic rumble, heard with the bell of the stethoscope over the apex is also present. The high-pitched opening snap (OS) is caused by the abrupt stopping of the domed mitral valve into the left ventricle (also appreciated in most patients midway between the left sternal border and apex). A shorter A2-OS distance indicates a more severe MS. Signs of pulmonary hypertension such as a loud P2 and right ventricular hypertrophy can also be present as MS becomes more severe. Mitral stenosis is defined as the reduced ability of the blood to move from the left atrium to the left ventricle in diastole. It is mostly caused by dysfunction in the mitral valve, which lacks the ability to open its leaflets in diastole. Mitral valve stenosis (MS) is predominantly caused by rheumatic carditis and is more prevalent in female patients. 00:0000:00 Physical signs of severe aortic regurgitation (A) include a rapid, quick arterial pulse, a wide pulse pressure, an early high-pitched, blowing diastolic murmur heard best over the left sternal border, an S3 gallop, and a low-pitched diastolic murmur at the apex (Austin-Flint murmur).The typical physical signs of severe aortic valve stenosis (B) are diminished carotid pulses (delayed and weak), a sustained apical impulse, a single second heart sound, an S4 gallop, and midsystolic crescendo-decrescendo murmur with late peaking best heard at the base of the heart. In mitral regurgitation (C) patients will display a systolic murmur, most often holosystolic, high-pitched and present at the apex with radiation to the axilla, left scapula, middle back, or left sternal border, depending on the direction of the regurgitant jet.

A 45-year-old woman presents to the ED complaining of one day of vision loss in her left eye associated with a left retro-orbital headache. She denies trauma or preceding illness. On exam, her vital signs are T 36.8ºC, BP 112/80 mm Hg, HR 74 bpm. Visual acuity is 20/20 in the right eye, 20/400 in the left eye, and 20/40 using both eyes. She has a left-sided afferent pupillary defect. Visual field testing reveals left-sided central vision loss. Which of the following is most accurate regarding her condition? Strict blood glucose control reduces the likelihood for future recurrences The diagnosis is confirmed by measurement of an intraocular pressure > 20 The diagnosis is made by magnetic resonance venography (MRV) The syndrome is associated with multiple sclerosis Therapy should be aimed at removing the acute obstruction of the ophthalmic artery

Correct Answer ( D ) Explanation: The patient's exam is consistent with optic neuritis, an acute monocular loss of vision caused by focal demyelination of the optic nerve. Most cases are retrobulbar and do not involve any visible changes on fundoscopy (especially during an acute episode). But, on occasion and with more protracted cases, optic disk pallor may be seen. An afferent pupillary defect, however, is always present. Approximately 30% of patients presenting with acute optic neuritis develop multiple sclerosis within 5 years. The classic clinical syndrome of MS consists of recurring episodes of neurologic symptoms that rapidly evolve over days and slowly resolve. Controlling blood glucose levels (A) has not been shown to influence the course of optic neuritis or multiple sclerosis. Optic neuritis is not associated with (B) increased (i.e., > 20) intraocular pressure. Magnetic resonance venography (MRV) (C) does not have a role in the diagnosis of optic neuritis or MS. However, a T2-weighted MRI can aid in the diagnosis by demonstrating lesions in the periventricular white matter—a finding that differentiates simple optic neuritis (where white matter changes are absent) from MS. Such an approach is critical in restoring vision in cases of central retinal artery occlusion (E), not optic neuritis.

A 5-year-old girl presents to the ED with a rash that started on her face and spread to her neck, axillae, and groin. Mom states that the patient had an upper respiratory infection one week prior. On examination, the patient's rash is tender to the touch. Which of the following statements regarding the diagnosis of this patient's condition is correct? Deep layers of the dermis are involved It often leaves the patient disfigured from scarring Mucous membrane involvement is common The disease is caused by an exotoxin-producing bacterium

Correct Answer ( D ) Explanation: This patient has staphylococcal scalded skin syndrome (SSSS), which is caused by an exotoxin-producing strain of Staphylococcal aureus. SSSS is actually a severe form of bullous impetigo. It is usually seen in children <5 years of age and often follows a URI. The rash usually starts on the face (perioral is classic), neck, axillae, and groin. The rash then spreads, becomes exfoliative, and is followed by the development of flaccid bullae and skin desquamation. The rash is characteristically tender to the touch, and gentle traction on the affected skin produces peeling (positive Nikolsky's sign). Unlike Stevens-Johnson syndrome, mucous membranes are not (C) involved in SSSS. The cleavage plane is intraepidermal; therefore, only superficial layers of the epidermis (A) shed and usually resolve in two weeks without scarring (B).

A 73-year-old woman with a history of hypertension presents with a unilateral headache for 3 weeks. She states that she has a throbbing pain at her right temple and has pain in her jaw with opening and closing. The vision in her right eye has worsened over the previous day. Her blood pressure is 173/100. What management is indicated? Carbamazepine Labetalol Non-contrast head CT scan Prednisone

Correct Answer ( D ) Explanation: This patient presents with a unilateral, subacute headache with associated jaw claudication and vision change; symptoms consistent with temporal arteritis. Temporal arteritis or giant cell arteritis is a systemic inflammatory process of small and medium-size arteries. The most commonly involved vessels are the ophthalmic vessels and the extracranial branches of the aortic arch. The disease typically affects patients over 70 years of age and is more common in women than in men. Patients present with a subacute headache that is throbbing in nature and may be present for weeks to months. Often, patients will have symptoms for more than 2 months. Patients may also report jaw claudication secondary to vascular insufficiency of the masseter and temporalis muscles. Physical examination may reveal tenderness over the temporal artery. Systemic symptoms may also be present including fever, joint pains, and weight loss. Diagnostic testing in the Emergency Department generally begins with an erythrocyte sedimentation rate (ESR) with a cutoff of 50 mm/hour although the level may be >100 mm/hour. However, the ESR will be normal in 10-25% of patients. The gold standard diagnostic test is a temporal artery biopsy. In patients with a high-clinical likelihood of temporal arteritis, treatment should be initiated regardless of initial diagnostic testing as delay can lead to permanent visual loss. Prednisone should be started at 60 - 120 mg/day. Carbamazepine (A) is the treatment of choice for trigeminal neuralgia, not temporal arteritis. The patient does not present with symptoms consistent with hypertensive emergency requiring emergent antihypertensive treatment with labetalol (B). A non-contrast head CT scan (C) is not helpful in temporal arteritis as the disease does not involve the intracranial contents.

A 19-year-old G1P0 woman at 26 weeks presents with abdominal pain after being involved in a motor vehicle collision. External pelvic examination reveals vaginal bleeding. Which of the following is true regarding this presentation? A normal ultrasound rules out placental abruption Disseminated intravascular coagulation is uncommon in placental abruption Early pelvic digital examination should be performed Emergent fetal monitoring and obstetric consultation are required

Correct Answer ( D ) Explanation: This patient presents with painful vaginal bleeding in late pregnancy after a trauma concerning for placental abruption and should have emergent fetal monitoring and obstetric consultation. Late pregnancy vaginal bleeding is never normal and raises suspicion for placental abruption, placenta previa and vasa previa; all potentially life threatening disorders to both the mother and fetus. Abruption occurs when there is separation of the placenta from the uterine wall leading to hemorrhage. It is more common in women with hypertension, preeclampsia, smoking, thrombophilia, cocaine use, trauma and prior abruption. Although most patients with abruption will present with dark vaginal bleeding, there are patients who will have hidden hemorrhage in which bleeding is contained between the placenta and the uterus. In patients with late term vaginal bleeding, emergent obstetric consultation should be obtained to further investigate and differentiate the causes. Fetal monitoring should be performed as distress may prompt emergent delivery depending on gestational age. Disseminated intravascular coagulation (DIC) (B) is relatively common and should always be considered. A digital examination (C) should be deferred until placenta previa is ruled out as digital examination may lead to worsening hemorrhage in this disorder. Ultrasound (A) should be obtained early in management to help to identify placenta previa and placental abruption but it is insensitive as the echogenicity of fresh blood is difficult to distinguish from the placenta. Therefore, a normal ultrasound does not rule out abruption.


संबंधित स्टडी सेट्स

Fluid regulation, water, electrolytes

View Set

Comptia N10-007 Practice test Questions Part 2

View Set

Med Surg Success Musculoskeletal Practice Test

View Set